You are on page 1of 55

Content downloaded/printed from HeinOnline

Thu Nov 21 07:11:13 2019

Citations:

Bluebook 20th ed.


Robert M. Jarvis, An Anecdotal History of the Bar Exam, 9 Geo. J. Legal Ethics 359
(1996).

ALWD 6th ed.


Robert M. Jarvis, An Anecdotal History of the Bar Exam, 9 Geo. J. Legal Ethics 359
(1996).

APA 6th ed.


Jarvis, R. M. (1996). An anecdotal history of the bar exam. Georgetown Journal of
Legal Ethics, 9(2), 359-412.

Chicago 7th ed.


Robert M. Jarvis, "An Anecdotal History of the Bar Exam," Georgetown Journal of Legal
Ethics 9, no. 2 (Winter 1996): 359-412

McGill Guide 9th ed.


Robert M Jarvis, "An Anecdotal History of the Bar Exam" (1996) 9:2 Geo J Leg Ethics
359.

MLA 8th ed.


Jarvis, Robert M. "An Anecdotal History of the Bar Exam." Georgetown Journal of Legal
Ethics, vol. 9, no. 2, Winter 1996, p. 359-412. HeinOnline.

OSCOLA 4th ed.


Robert M Jarvis, 'An Anecdotal History of the Bar Exam' (1996) 9 Geo J Legal Ethics
359

-- Your use of this HeinOnline PDF indicates your acceptance of HeinOnline's Terms and
Conditions of the license agreement available at
https://heinonline.org/HOL/License
-- The search text of this PDF is generated from uncorrected OCR text.
-- To obtain permission to use this article beyond the scope of your license, please use:
Copyright Information

Use QR Code reader to send PDF to your smartphone or tablet device


An Anecdotal History of the Bar Exam
ROBERT M. JARVIS*

Although it is a rite of passage that nearly every lawyer has endured, few
know very much about the history of the bar exam. This lack of knowledge is
largely due to the fact that very little has been written about the bar exam's
rise from humble formality to career-threatening ordeal. The goal of this
Article is to shed some light on one of life's most dreaded experiences.
Because nothing comes easily when one is dealing with the bar exam, this
Article does not simply place its hard-won wisdom before its readers.
Rather, like the bar exam itself, it is divided into two distinct parts. Part I
consists of twenty-five (25) multiple choice questions on the evolution of bar
exams. Part II includes three (3) essay questions on the contemporary
administration of bar exams. The right answers appear after the last essay
question..
Finally, as will be seen, there are no footnotes in this Article except for
the one containing the author's biographical information. That is because
footnotes never appear in bar exams.

I. HISTORICAL DEVELOPMENT

1. In order to represent oneself in court, an individual must:


(a) have matriculated at a law school approved by the American Bar
Association.
(b) be of legal age.
(c) have graduated from a law school recognized by the state supreme
court.
(d) have attended law school for at least one year.
2. If a person chooses to "read law," he or she:
(a) does not have to graduate from law school.
(b) does not have to take the bar exam.
(c) does not have to join the state bar association.
(d) it is no longer possible to read law in the United States.

* Professor of Law, Nova Southeastern University. B.A., Northwestern University; J.D., Univer-
sity of Pennsylvania; LL.M., New York University. A modified version of this Article will appear as a
chapter in the forthcoming book, History of Legal Education in the United States, to be published by
Salem Press, Inc. of Pasadena, California.
GEORGETOWN JOURNAL OF LEGAL ETHICS [Vol. 9:359

3. The Baby Bar Exam:


(a) is the nickname given to the separate bar exam taken by lawyers who
wish to practice family or juvenile law.
(b) is a simulated bar exam that is taken approximately three weeks before
the actual bar exam.
(c) exists only in California.
(d) refers-to the national General Legal Skills Exam taken by all law
school students immediately prior to graduation.

4. Upon graduating from law school, a person:


(a) is automatically entitled to practice law in all fifty states.
(b) receives the degree of Juris Doctor.
(c) is considered a lawyer.
(d) is admitted to practice in the state in which his or her law school is
located.

5. Bar/Bri is an abbreviation for:


(a) Bay Area Review/Bar Review Inc.
(b) Barristers Ball of Rhode Island, the party given each October by the
Rhode Island Bar Association to honor recent bar admittees.
(c) Board of Attorney Reviews, Bans and Reinstatements of Iowa.
(d) Bar/Bar-Related Institute, the national association of bar exam
executives.

6. Bar exams originally were oral. Where and when was the first written bar
exam given?
(a) In New York in 1682.
(b) In Georgia in 1794.
(c) In Massachusetts in 1855.
(d) In Michigan in 1931.

7. Which state was the first to establish a permanent, full-time board of bar
examiners financed by applicant fees?
(a) Nevada (1907).
(b) New Hampshire (1880).
(c) Kentucky (1853).
(d) Maine (1899).
1996] ANECDOTAL HISTORY OF THE BAR EXAM

8. According to Jonathan M. Birch, an Illinois lawyer, Abraham Lincoln


was a(n) - bar examiner:
(a) fair
(b) impossibly demanding
(c) lenient
(d) prudish

9. Huey P. Long, who later served as governor of Louisiana, passed his bar
exam:
(a) while working full-time as a page in the Mississippi House of Represen-
tatives.
(b) by promising to refer business to a member of his examining commit-
tee.
(c) with the highest score ever obtained by any person, before or since, in
Louisiana.
(d) in just 53 minutes, the fastest recorded time in Louisiana's history.

10. In which states must an applicant wear a suit-and-tie or dress (as may be
appropriate) while taking the bar exam?
(a) Arizona and California.
(b) Kentucky and Virginia.
(c) New Jersey and New York.
(d) None of the above.

11. Until 1933, in which state could citizens practice law without having to
go to law school or take the bar exam?
(a) Maine.
(b) Tennessee.
(c) Indiana.
(d) South Carolina.

12. The Multistate Bar Examination is:


(a) a national bar exam that permits successful candidates to practice in
any state.
(b) a regional bar exam used by groups of states known as "cluster" states.
(c) a national bar exam that was proposed by Chief Justice Earl Warren
but never adopted.
(d) prepared and scored by the National Conference of Bar Examiners.
GEORGETOWN JOURNAL OF LEGAL ETHICS [Vol. 9:359

13. Which of the following subjects is not tested on the Multistate Essay
Examination?
(a) Conflicts of Law.
(b) Bankruptcy.
(c) Corporations.
(d) Wills and Trusts.

14. The Multistate Professional Responsibility Examination:


(a) is designed to determine the trustworthiness of bar applicants.
(b) has a passing rate of 90% in many states.
(c) must be taken by all lawyers every five years.
(d) is a comprehensive exam that includes, in addition to ethics, such
subjects as jurisprudence, philosophy and sociology.

15. The California performance examination is designed to:


(a) predict whether a particular bar applicant will commit malpractice
during his or her legal career.
(b) ensure continuing lawyer competency by subjecting practicing lawyers
to retesting every three years.
(c) determine an applicant's knowledge of community property law.
(d) test an applicant's lawyering skills.

16. Candidates who do well on the bar exam:


(a) make more money than other lawyers.
(b) do more pro bono work than other lawyers.
(c) are more likely to be named to the bench than other lawyers.
(d) there is no correlation between how one does on the bar exam and
how one does in practice.

17. Which of the following events never happened?


(a) A woman dressed to look like a man took the bar exam for her
husband.
(b) A bar exam was postponed because examinees insisted that they had
seen UFOs landing on the roof.
(c) A bar applicant complained that an essay question contained sexual
language that was too graphic.
(d) Bar exam answers were thrown out before they were graded.
19961 ANECDOTAL HISTORY OF THE BAR EXAM

18. While serving as the President of the American Bar Association in 1991,
Talbot "Sandy" D'Alemberte proposed:
(a) that bar exams be eliminated.
(b) that bar exams be made tougher.
(c) that students be allowed to take bar exams while still in law school.
(d) that bar exams include "topics that particularly affect women, such as
abortion, rape and domestic violence."
19. In order to be admitted to the bar, a person must:
(a) presently live in the state in which admission is sought.
(b) plan to live in the state in which admission is sought.
(c) have once lived in the state in which admission is sought.
(d) none of the above.
20. "Bridge-the-Gap" programs are designed to:
(a) help college students make the transition to law school.
(b) help new lawyers make the transition to legal practice.
(c) help old lawyers make the transition to retirement.
(d) all of the above.
21. After a person is admitted to the bar, he or she:
(a) must practice for one year under the supervision of an experienced
attorney.
(b) must practice for two years under the supervision of an experienced
attorney.
(c) must handle, or "co-counsel," cases with an experienced attorney for
five years.
(d) generally speaking, is free to take cases without any supervision.
22. Before a lawyer can use the title "counselor-at-law," he or she must:
(a) have been in practice for at least five years.
(b) have handled a minimum of fifty cases or matters.
(c) have taken and passed the counselor's exam.
(d) have been admitted to the bar.
23. Once a person has been admitted to practice in one jurisdiction, he or
she:
(a) may ask to appearpro hac vice in other jurisdictions.
(b) can practice in any other jurisdiction.
364 GEORGETOWN JOURNAL OF LEGAL ETHIcs [Vol. 9:359

(c) is eligible to take the bar exam in any other jurisdiction.


(d) will be admitted on motion in any other jurisdiction.
24. In order to be admitted to the United States Supreme Court, a person
must:
(a) be nominated by a justice of the Court.
(b) take and pass the United States Supreme Court Attorneys' Exam.
(c) have been admitted to the highest court of a state or territory for at
least three years.
(d) have argued a minimum of twenty-five cases in the lower federal
courts.
25. A lawyer from a foreign country will be permitted to take the bar exam
in the United States:
(a) assuming that he or she meets all of the requirements specified in the
1987 United Nations Convention on the Recognition of Foreign Law
Degrees and Law Licenses ("the Vienna Treaty").
(b) on the same basis as an American citizen.
(c) if the lawyer comes from a country whose legal system is based on the
common law.
(d) a foreign country lawyer cannot practice law in the United States
because only American citizens can take the Attorney's Oath.

II. CURRENT USAGE

A.
Larry Harris recently graduated from a small, unaccredited law school.
When he applied to take his state's bar exam, he was told that he was not
eligible to do so because he had not graduated from a law school approved
by the American Bar Association. His application also was rejected because
Harris lacks a college degree. Harris has asked you to help him in his quest
to take the bar exam. Please advise him as to his chances.

B.
Lisa Smith, a black woman, has twice failed her state's bar exam. Under
the rules of her state, individuals may sit for the bar three times; thereafter,
they are permanently ineligible to take the test. Smith has asked you to file
suit against the bar examiners on the following grounds: 1) the limit on the
number of times a person can take the bar exam is unconstitutional; 2) the
examiners' refusal to release copies of their questions, model answers and
scoring processes is improper; 3) the test in its present form is culturally
1996] ANECDOTAL HISTORY OF THE BAR EXAM

biased and, as a result, is unfair to African-Americans; and, 4) the current


pass rate, which has hovered around 60 percent for the past five years,
proves that the examiners are intentionally and illegally limiting the number
of new attorneys. Please advise her of her likelihood of success.

C.
Roger North has been denied admission to the bar in his home state.
Although he successfully passed all parts of the bar exam, the bar examiners
concluded that North lacked the character and fitness required to become a
lawyer. The examiners based their decision on the fact that North: formerly
had a serious drug habit (he has since recovered and now is a counselor in a
local drug rehabilitation program), has been under the care of a psychiatrist
for the past ten years (he continues to see his doctor once a week), has twice
discharged his debts through bankruptcy proceedings and was prosecuted
many years ago for allegedly sexually abusing his former fiancee (the
charges were dropped before trial).
North revealed all of the foregoing incidents on his initial bar application.
He believes that his past mistakes should not be held against him and has
asked you to help him gain admission to the bar. Please advise him whether
he is likely to be successful.

III. ANSWERS TO MULTIPLE CHOICE QUESTIONS


1. (b) Every natural person who is of legal age has the right to appear in
court without an attorney regardless of his or her lack of legal knowledge or
training. See Cheung v. Youth OrchestraFound., Inc., 906 F.2d 59, 61 (2d Cir.
1990) ("The right to conduct one's own litigation 'is a right of high standing
.....' "); United States v. Dougherty, 473 F.2d 1113, 1128 (D.C. Cir. 1972) ("A
defendant has the moral right to stand alone in his hour of trial.").
Accordingly, (a), (c) and (d) are wrong.
Persons who choose to represent themselves are known asprose litigants.-
(The more formal in propriapersona has fallen into general disuse.) The
right to proceed pro se is a historical one that dates to the founding of the
Republic. See 28 U.S.C. § 1654 (1988) ("In all courts of the United States
the parties may plead and conduct their own cases personally or by
counsel.... ."). Because the right extends only to natural persons, an
attorney must represent all other litigants, including corporations and
partnerships. See Eagle Assocs. v. Bank of Montreal, 926 F.2d 1305, 1310 (2d
Cir. 1991) (holding that 28 U.S.C. § 1654 prohibits a layperson from
appearing on behalf of a partnership); Expressway Assocs. II v. Friendly Ice
Cream Corp., 642 A.2d 62, 64 (Conn. App.) (holding that a corporation may
not appearpro se), appeal denied, 645 A.2d 1018 (Conn. 1994).
GEORGETOWN JOURNAL OF LEGAL ETHICS [Vol. 9:359

At one time, many courts refused pro se requests by criminal defendants


due to the belief that the Sixth Amendment required such persons to have
counsel. In Faretta v. California, however, the U.S. Supreme Court ruled
that the criminally accused can represent themselves so long as they make a
"knowing[] and intelligent[]" waiver of their constitutional rights. 422 U.S.
806, 835 (1975). See also State v. Panzera, 652 A.2d 136, 138 (N.H. 1994)
(finding that the right to represent oneself in a criminal trial is guaranteed
by both federal and state law).
The courts accord no special treatment to persons who decide to act as
their own attorneys and hold them to the same standards as attorneys
representing parties. See Dufort v. Bangs, 644 A.2d 6, 7 (Me. 1994) (sanction-
ingpro se litigants for frivolous appeals); Gaub v. Simpson, 866 P.2d 765, 766
n.1 (Wyo. 1993) (applying rules of appellate procedure to pro se litigants).
Pro se litigants do not, however, have to demonstrate the same mastery of
pleading and procedure as lawyers. See Kincaid v. Vail, 969 F.2d 594, 598
(7th Cir. 1992) (applying less stringent standards toprose litigants respond-
ing to summary judgement motions), cert. denied, 113 S. Ct. 1002 (1993);
Balistreriv. Pacifica Police Dep't, 901 F.2d 696, 699 (9th Cir. 1988) (liberally
construingpro se litigant's appellate brief).
The subject of pro se representation is considered further in Alicia L.
Downey, Note, Fools and Their Ethics: The ProfessionalResponsibility of Pro
Se Attorneys, 34 B.C. L. Rev. 529 (1993) (urging adoption of a new Model
Rule onpro se attorneys); Helen B. Kim, Note, Legal Educationfor the Pro Se
Litigant:A Step Towards a Meaningful Right to be Heard, 96 Yale L.J. 1641
(1987) (proposing classroom education forpro se litigants as an alternative
to attorney representation); Joseph M. McLaughlin, Note, An Extension of
the Right of Access: The Pro Se Litigant's Right to Notification of the Require-
ments of the Summary Judgment Rule, 55 Fordham L. Rev. 1109 (1987)
(arguing for pro se litigants' greater accessibility to courts); Eric J.R.
Nichols, Note, PreservingPro Se Representation in an Age of Rule 11 Sanc-
tions, 67 Tex. L. Rev. 351 (1988) (examining application of Rule 11 to pro se
parties).

2. (a) "Reading law" (also referred to as "law office study") was the system
by which a person learned the law under the tutelage of a practicing lawyer,
usually by working in that lawyer's office as a clerk:
The conventional arrangement was for aspiring attorneys to offer their
services plus a fee to established lawyers in exchange for instruction and
the right to use their masters' forms later in practice. In an era before
typewriting, preprinted forms, and duplicating services, many students
spent most of their apprenticeships copying writs and documents.
Deborah L. Rhode & David Luban, Legal Ethics 810-11 (2d ed. 1995). See
1996] ANECDOTAL HISTORY OF THE BAR EXAM

also Charles R. McKirdy, The Lawyer as Apprentice: Legal Education in


Eighteenth Century Massachusetts, 28 J. Legal Educ. 124 (1976) (describing
the legal clerkship system in colonial America).
Before the advent of law schools, reading law was the means by which
most persons became lawyers in the United States. Those with the financial
resources to do so, however, often opted to travel to London to study at one
of the English inns of court. See Marian C. McKenna, Tapping Reeve and
The Litchfield Law School 6 (1986) (noting that South Carolina sent more of
its citizens to England than any other colony). Even after law schools were
well-established, reading law remained a popular alternative; as late as
1950, thirty-five states allowed law office practice in lieu of law school.
George N. Stevens, Diploma Privilege, Bar Examination or Open Admission,
46 B. Examiner 15, 15 (1977).
Today, only seven states (California, Maine, New York, Vermont, Vir-
ginia, Washington and Wyoming) permit aspirants to read law and only a
handful of individuals elect to do so. Mark Curriden, Lawyers Who Skip Law
School, A.B.A. J., Feb. 1995, at 28; accord Sarazin v. Vermont Board of Bar
Examiners, 639 A.2d 71, 71 n.1 (Vt. 1994) (noting that Vermont, California,
Virginia and Washington allow attorneys to read law); Michael D. Shear,
'Law Readers' Take an Unusual Road to the Va. Bar, Wash. Post, Aug. 10,
1994, at B1 (describing reading law in Virginia). Accordingly, (d) is wrong.
Those who read law must still take the bar exam and join the state bar
association (where doing so is a requirement for licensure). See Mark
Curriden, Lawyers Who Skip Law School, A.B.A. J., Feb. 1995, at 28 (noting
that states with "reading law programs" still require people to pass bar
exams to obtain licenses). Thus, (b) and (c) are wrong.
As has been pointed out elsewhere, the usefulness of law office study
depended entirely on the skill, interest, knowledge and enthusiasm of the
supervising lawyer. See Charles R. McKirdy, The Lawyer as Apprentice: Legal
Education in Eighteenth Century Massachusetts, 28 J. Legal Educ. 124, 134
(1976) ("[Most students'] individual experiences were as varied as the men
who trained them."). As such, law-office study ran the gamut "from an
important educational experience to gross exploitation." Robert Stevens,
Law School: Legal Education in America from the 1850s to the 1980s 3 (1983).
One student who apparently benefitted from reading law was John Quincy
Adams, who wrote of his preceptor:
It is of great advantage of us to have Mr. [Theophilus] Parsons in the
office. He is in himself a law library, and proficient in every useful branch
of service; but his chief excellency is, that no student can be more fond of
proposing questions than he is of solving them. He is never at a loss, and
always gives a full and ample account, not only of the subject proposed,
but of all matters which have any intimate connection with it. I am
persuaded that the advantage of having such an instructor is very great.
GEORGETOWN JOURNAL OF LEGAL ETHIcs [Vol. 9:359

Id. at 10 n.5 (citation omitted). In contrast, Adams' father, John Adams, was
highly critical of his tutor and "there is evidence that [the elder] Adams did
not read all the assignments." Id. (citing Gerard W. Gawalt, The Promise of
Power: The Emergence of the Legal Profession in Massachusetts 1760-1840
132, 135 (n.d.)).
In addition to the lack of quality control, it has been argued that the law
office study system "induced a rigid, unquestioning attitude toward legal
studies... [and] impeded the legal system's capacity to respond to changes
occurring in society in general." See 1 The Papersof Daniel Webster 4 (Alfred
S. Konefsky & Andrew J. King eds., 1982). Recently, however, the sound-
ness of this widely held view has been questioned. See Craig E. Klafter, The
Influence of Vocational Law Schools on the Origins of American Legal
Thought, 1779-1829, 37 Am. J. Legal Hist. 307, 331 (1993) (concluding that
the system "instilled in [its] graduates a conception of the law as a dynamic
institution constantly improving as it is encouraged to adapt to the needs of
a changing society").

3. (c) The Baby Bar Exam, formally known as the First Year Law Student
Examination (FYLSX), is a unique California institution. Since 1935, all
students attending unaccredited California law schools have had to take the
FYLSX following their first year of law school; only those who pass the test
are permitted to later sit for the General Bar Exam. See Bib'le v. Committee
of Bar Examiners, 606 P.2d 733, 734 (Cal.) (concluding that the petitioner
had not completed the FYLSX requirement), cert. denied, 449 U.S. 860
(1980).
The FYLSX was created in response to recruiting abuses by unaccredited
law schools and the frequently expressed concern that such schools "ad-
vance all students, regardless of ability or performance." Lupert v. California
State Bar, 761 F.2d 1325, 1328 n.3 (9th Cir.), cert. denied, 474 U.S. 916
(1985). Although the FYLSX has been challenged on equal protection
grounds, it has been held to be constitutional because it advances the
important state interests of "protect[ing] persons from continuing to pursue
a profession for which they are not qualified, and aid[ing] qualified persons
[to] judg[e] the quality of the training and education they are receiving in
preparation for the bar examination." Id. at 1328-29. For a further discus-
sion of the FYLSX, see Stephen G. Hirsch, Non-Lawyer ShinesAs Chief Bar
Examiner, Recorder, Jan. 22, 1991, at 1 (discussing fairness of forcing only
students at unaccredited schools to take the FYLSX).
(a) is wrong because a separate bar exam does not exist for those who
wish to practice family or juvenile law. Many states, however, have estab-
lished certification programs in these areas; those who become certified are
permitted to hold themselves out as specialists. See, e.g., Business Briefs,
Times-Picayune, May 28, 1995, at 19H3 (noting Louisiana's certification
1996] ANECDOTAL HISTORY OF THE BAR EXAM

program in family law). (b) is wrong because there is no standard simulated


bar exam. (d) is wrong because, unlike some disciplines, there is no
comprehensive final exam that all law students must take.

4. (b) The basic degree in law currently awarded in the United States is the
Juris Doctor or J.D. degree. Once known as the LL.B. (for Bachelor of Laws
and Letters), it has been replaced in all law schools by the J.D.
After receiving the J.D., some students, particularly those interested in
highly specialized fields (such as taxation) elect to pursue an LL.M. (Master
of Laws and Letters). Ken Myers, Students Tapping LL.M. Programs to
'Master' Their Own Fates, Nat'l L.J., July 31, 1995, at A16. A rare few
(primarily those interested in an academic career) go further and earn the
S.J.D. (sometimes called the J.S.D.), the Doctor of Juridical Science. The
S.J.D. is the law's equivalent to a Ph.D. and is the terminal degree in law.
Federal Reports, Inc., Directory of Graduate Law Degree Programs xii
(Richard L. Hermann et al. eds., 3d ed. 1992). In addition to the S.J.D., law
schools also award the honorary LL.D. (Doctor of Laws and Letters) to
dignitaries, celebrities and major donors. See, e.g., Robert B. Charles, Legal
Education in the Late Nineteenth Century, Through the Eyes of Theodore
Roosevelt, 37 Am. J. Legal Hist. 233, 239 n.43 (noting that Theodore
Roosevelt received many such degrees during his life from such schools as
Harvard and Yale). The classic study of the development of the first
American law degrees during the late nineteenth century is Alfred Z. Reed,
Trainingfor the Public Profession of the Law 160-81 (1976).
Although it is commonly believed that graduating from law school
automatically entitles an individual to practice law throughout the country,
this assumption is erroneous. Thus, (a) is wrong. Likewise, (c) is wrong. A
lawyer is a person who has been admitted to practice in at least one
jurisdiction.
(d) is wrong because it is only a partially correct answer. At one time,
graduates of many law schools were automatically admitted to the bar of the
state in which their law school was located by virtue of a statutory exemption
from the bar exam known as "diploma privilege." Alfred Z. Reed, Training
for the Public Profession of the Law 248-53 (1976). In 1892, however, the
American Bar Association began a concerted campaign to have the privi-
lege abolished. See David M. White, The Definition of Legal Competence:
Will the Circle be Unbroken?, 18 Santa Clara L. Rev. 641, 659 n.84 (quoting
Robert Stevens, Two Cheersfor 1870: The American Law School, in Law in
American History 405, 458 (Donald Fleming & Bernard Bailyn eds., 1971)
(volume five of the Perspectives in American History series)). As a result, it
exists today only in Wisconsin (where it benefits graduates of Marquette
University and the University of Wisconsin). See State ex rel. Quelch v.
Daugherty, 306 S.E.2d 233, 238 (W. Va. 1983) (quoting Law & Business,
GEORGETOWN JOURNAL OF LEGAL ETHICS [Vol. 9:359

Inc., Minimum Requirements for Admission to Legal Practice in the United


States, Lawyer's Almanac 1982-83, at 202 (1982)).
The arguments for and against the exemption are canvassed in George N.
Stevens, Diploma Privilege, Bar Examination or Open Admission, 46 B.
Examiner 15 (1977). Compare State ex rel. Quelch v. Daugherty, 306 S.E.2d
233 (W. Va. 1983) (expressing strong support for the privilege) with In re
ProposedAmendments Concerningthe BarExamination,609 P.2d 263 (Mont.
1980) (describing the privilege as both outdated and unfair to graduates of
out-of-state law schools).

5. (a) Bar/Bri, the largest and best-known bar review course in the country,
was created in 1974 when the publishing giant Harcourt Brace Jovanovich
spent several million dollars to acquire Bay Area Review, a California-
based bar review course, and Bar Review Inc., a Chicago-based bar review
course, and then merged the two companies into one. Kenneth B. Noble,
The Bar-Review Course: HighAnxiety, Big Business, N.Y. Times, Jan. 4, 1981,
at 10; see also James Warren et al., They Get to Become Lawyers -And
Conviser Gets Rich, Chi. Trib., June 25, 1985, at C1 (describing Bar/Bri's
courses in Illinois); Richard J. Conviser, Of Dreams and Cathedrals: A
Tribute to Lewis Collens, 65 Chi.-Kent L. Rev. 659, 662 (1989) (describing
Bar/Bri as "one of the nation's largest legal educational companies"). (b),
(c) and (d) are wrong because each describes a mythical organization or
event.
At one time, new law school graduates prepared themselves for the bar
exam by studying on their own or, in some instances, attending ad hoc
lectures given by a local lawyer or judge. Typical of these early instructional
efforts were those of Judge Harold R. Medina, Jr. After graduating from
Columbia Law School, Medina began preparing bar applicants for a fee of
$35. By the time he stopped in 1940, it was said that he had tutored almost
40,000 applicants. J.Y. Smith, Harold R. Medina, 102, Dies; Ran 1949
Conspiracy Trial, Wash. Post, Mar. 17, 1990, at B5.
One problem that Medina and others like him faced was the fact that they
had to become and remain masters of all of the subjects that possibly could
appear on the bar exam. See Ralph Brill, A Tribute to Professor Warren
Heindl Upon His Retirement, 69 Chi.-Kent L. Rev. 843, 843 (1994) (noting
that the Thomas Hardy Bar Review Course of Illinois, the predecessor to
Bar/Bri, was "a one man operation - Hardy did all the outlines and
lectures himself"). Because of this fact, it was impossible for even the most
ambitious of entrepreneurs to expand much beyond their home states.
The advent of standardized bar exam testing in 1972, however, coupled
with a burgeoning law student population, increased lawyer mobility, and a
growing acceptance of mass marketing advertising techniques, transformed
bar review courses into sophisticated, for-profit, multi-jurisdictional enter-
1996] ANECDOTAL HISTORY OF THE BAR EXAM

prises utilizing as many as 150 lecturers. Kenneth B. Noble, The Bar-Review


Course: High Anxiety, Big Business, N.Y. Times, Jan. 4, 1981, at 10. As a
result, bar review courses today constitute a $50 million-a-year industry that
serves more than 45,000 students. See Ken Myers, Jury Reviews Battle
Between Bar Courses, Awards $4 Million, Nat'l L.J., June 7, 1994, at A13
(noting that Bar/Bri has an estimated 35,000 customers annually in the $50
million-a-year bar review business). As for the solo operations, they mostly
have disappeared. An important exception is John Pieper, who continues to
offer a one-man bar review course in New York City. See Dan Hurley, The
Participantsat John Pieper's Bar Review Course Are So Enthusiastic, You'd
Swear You Were Going to a Pep Rally, Student Law., Dec. 1985, at 38
(describing Pieper's bar review course); David Margolick, 'Guru' of Law
Helps Students Conquer the Bar, N.Y. Times, July 16, 1984, at B1 (same).
Although Bar/Bri currently is the undisputed king of bar review courses,
remaining number one has not been easy.
In May 1994, for example, Bar/Bri was found guilty by a California jury of
having used illegal business practices against a competitor known as Ameri-
can Professional Testing Service Inc. (APTS). Ken Myers, Jury Reviews
Battle Between Bar Courses, Awards $4 Million, Nat'l L.J., June 7, 1994, at
A13. In May 1995, however, the trial judge set aside the verdict. Ken Myers,
With Purchase of Review Course, West Group Bellies Up to the Bar, Nat'l L.J.,
July 24, 1995, at A15.
In June 1994, Bar/Bri was forced to pay $53,000 to settle a suit filed
against it by the U.S. Department of Justice under Title III of the Ameri-
cans with Disabilities Act. According to the government, Bar/Bri had
violated the statute by refusing student requests for interpreters and Braille
materials. Laura Duncan, Bar Review Settles Bias Suit, Chi. Daily L. Bull.,
June 10, 1994, at 3.
In January 1995, Stanley D. Chess and Steven H. Levine, two of Bar/Bri's
longtime top executives, jumped to the West Publishing Company's newly
formed Professional Education Group with the intention of starting a rival
bar review course. Ken Myers, Mr. Chess Makes SurprisingMove: He Leaves
Bar/Bri and Goes West, Nat'l L.J., Feb. 13, 1995, at A18; Frances A.
McMorris, Brawl Erupts in the Bar-Review Business, Wall St. J., Feb. 23,
1995, at B2. See also Ken Myers, Chess Moves, Nat'l L.J., Mar. 6, 1995, at
A13 (noting Bar/Bri's suit against Chess for breach of fiduciary duties and
fraud); Ken Myers, New Review Suit, Nat'l L.J., Mar. 13, 1995, at A20
(noting West's countersuit against Bar/Bri for antitrust violations).
Since becoming the head of West's new venture, Mr. Chess has pursued
an aggressive marketing campaign that has sought to woo both students and
lecturers away from Bar/Bri. He also has had West purchase APTS. Ken
Myers, With Purchase of Review Course, West Group Bellies Up to the Bar,
Nat'l L.J., July 24, 1995, at A15. Despite these initiatives, it is still much too
GEORGETOWN JOURNAL OF LEGAL ETHICS [Vol..9:359

early to say whether West will succeed. When Emanuel Outlines, Inc., the
enormously popular law study aid company, attempted to enter the bar
review field in August 1987, it quickly found itself outgunned and folded
after just six weeks in business. Alexander Stifle, Second-LargestBar Review
Course is Closed Six Weeks After Purchase, Nat'l L.J., Nov. 2, 1987, at 4;
Daniel Wise, Defunct Bar Course Cleared After Probe by Consumer Dept.,
N.Y. L.J., Mar. 28, 1988, at 1.
Numerous other examples can be found of the cutthroat competition that
exists among bar review courses. See, e.g., Palmer v. BRG of Ga., Inc., 498
U.S. 46, 48 (1990) (holding illegal a market allocation agreement between
competing bar review providers); HarcourtBrace Jovanovich Legal & Profes-
sionalPublications,Inc. v. MultistateLegal Studies, Inc., 26 F.3d 948, 952 (9th
Cir. 1994) (holding advertisements that disparaged competitor's bar review
course violated the terms of a prior consent decree); Marino v. Josephson
BarReview Ctr. ofAmerica, Inc., 517 F. Supp. 674 (S.D.N.Y. 1981) (denying
injunction against use of videotapes that allegedly damaged reputation of
former bar review lecturer); Kallen v. Nexus Corp., 353 F. Supp. 33 (N.D. I11.
1973) (dismissing antitrust suit by students who took the only bar review
course offered in Illinois). See also Chris Downey, Marketing Exam Prep
Courses to Jittery First-YearStudents, N.Y. L.J., Nov. 14, 1990, at 1 (describ-
ing the use of first-year law student exam review courses as marketing tools
for bar review courses); Martha Middleton, N. C. BarReview Course: Two's a
Crowd?, 7 B. Leader 25 (1982) (reporting allegations that state bar associa-
tion's bar review course improperly implied ties to bar examiners and better
preparation than commercial course); Ken Myers, BarReview Course Com-
petitors Are Duking It Out in the Courts, Nat'l L.J., Aug. 5, 1991, at 4
(describing suit against Bar/Bri by two competitors alleging unfair competi-
tion, false advertising, fraud and defamation); Ken Myers, Bar Review Scrap
Gets NastierAs Providers Trade Accusations, Nat'l L.J., Feb. 11, 1991, at 4
(reporting that bar review provider was considering suing Bar/Bri for libel,
spreading false rumors and antitrust violations); David E. Rovella, Calif.
Course Wins a Skirmish In the Battle of the Bar Reviews, Nat'l L.J., Sept. 18,
1995, at A17 (reporting on the revival of an anti-trust suit against Bar/Bri by
competitor Multistate Legal Studies, Inc.).
Despite the intense competition among bar review courses, students tend
to lump them together and complain equally about their length, cost,
faculty, materials, location and approach:
A bar review course is taught in the seediest hotel in town' The
ballroom has massive chandeliers that creak directly overhead and have
one working bulb that flickers. There is no ventilation. Large pitchers of
iced water are provided "free," but there are no working toilets. The
folding, metal chairs have plastic seat pads that glue themselves to your
1996] ANECDOTAL HISTORY OF THE BAR EXAM

skin if you are wearing shorts. The long metal tables have white table
cloths decorated with cigarette holes. Far in the front of the room, where
they cannot be seen, are the lecturers who are "experts" in each area of
the law and who all try a different act to keep you awake.
The first session, you may be given a sample exam which everyone will
fail (otherwise, what incentive to continue the course and not get a pro
rata refund?). At the end of the course, you will take another sample exam
which you will also fail. But by then you can't get a refund.
What I liked about my bar review course was that lecturers would
preface their sessions with one of two phrases, either "Listen carefully
because this will definitely be on the exam," or "We'll go through this
quickly because it definitely won't be on the exam." Wonder of wonders,
my lecturers were dead wrong. For example, the lecturer on criminal law
insisted that degrees of crimes would never be on the essay section or
short answer part of the state exam. He was right about the latter and
wrong about the former. The first essay question asked for an analysis of a
situation involving a variety of degrees.
As far as his being right about the short answer part, that was because
the lecturer on domestic relations had insisted that Dom. Rel. would
never be on the exam, and, of course, the short answer part was sixty
percent Dom. Rel. questions. Thank God I had just been through a
divorce.
By the way, the seedy setting for the bar review is the only aspect
relevant to the exam itself. Bar exams are held in condemned warehouses,
condemned buildings, on wharves, anywhere students will have to fear for
their lives getting there and home.

Laurens R. Schwartz, What You Aren't Supposed to Know About the Legal
Profession: An Expose of Lawyers, Law Schools, Judges and More ... An
Insider's Report 43 (1988). Even with their shortcomings, however, most
students are grateful that commercial bar review courses exist:

You're a fool if you don't sign up for one of these courses.... I have no
idea where you'd start if you wanted to do it on your own. There really are
patterns to the bar exam's questions, and these review courses can detect
them. They also give you mnemonic devices and shortcuts to cram your
memory full of the trivia on which you're likely to be tested. With their
assistance and a lot of devoted effort, you can make it.

Raymond L. Woodcock, Take the Bar and Beat Me 120-21 (1991). See also
Edward A. Adams, A Blizzard of Questions Heralds the BarExam, N.Y. L.J.,
July 24, 1990, at 1 (describing how BAR/BRI soothes nervous students in
the days leading up to the bar exam); Brenda Sapino, Playingthe BarReview
Game, Tex. Law., May 20, 1991, at 1 (reporting that in a survey of Texas law
students, most respondents were very satisfied with their bar review course).
GEORGETOWN JOURNAL OF LEGAL ETHICS [Vol. 9:359

6. (c) In 1855, Massachusetts became the first state to have a written bar
exam. Charles W. Wolfram, Modem Legal Ethics § 5.3, at 198 (1986).
Accordingly, (a), (b) and (d) are wrong.
Originally, most bar exams were conducted orally, either before a judge
of the court to which admission was sought or by one or more lawyers
already admitted to the court. Id.
At the interview the candidate was expected to prove that he or she
possessed both the legal knowledge and moral character necessary to be a
lawyer. Although most interviews were perfunctory affairs that were more
concerned with the applicant's good fellowship than with his or her legal
prowess, some were quite rigorous. See Barbara A. Babcock, Clara Short-
ridge Foltz: "First Woman", 30 Ariz. L. Rev. 673, 697 (1988) (describing the
three-hour oral exam of Clara Foltz, the first woman to be admitted to the
California bar); J. Clay Smith, Jr., Justice and Jurisprudenceand the Black
Lawyer, 69 Notre Dame L. Rev. 1077, 1078-79 (1994) (imagining the oral
exam of Macon B. Allen, the first African-American lawyer in the United
States).
The movement from oral to written exams began with the Massachusetts
Court of Common Pleas. Between 1855 and 1859, it required candidates
who could not show three years of legal study to pass a written exam. Alfred
Z. Reed, Trainingfor the Public Profession of the Law 101 n.3 (1921). In 1876,
the Suffolk County (Massachusetts) Board revived the practice of requiring
applicants to pass a written exam. In 1877, the New York State Supreme
Court introduced an exam that included both a written and an oral
component. Id. at 357 n.4. Within a short time, Idaho and Nevada also were
experimenting with written tests. Id. at 101 n.3.
Because the early written exams demanded only rote learning and basic
literary skills, they failed to function as effective tests of competence. See
Deborah L. Rhode & David Luban, Legal Ethics 814 n.3 (2d ed. 1995)
(citing Esther L. Brown, Lawyers and the Promotion of Justice 117 (1938)
(noting that eighty to ninety percent of all applicants eventually passed)).
Nevertheless, they were viewed as being both "technocratic" and "egalitar-
ian" and soon gained in popularity. Robert Stevens, Law School: Legal
Education in America from the 1850s to the 1980s 25-26 (1983). Today, no
state uses an oral bar exam, although such tests are starting to make a
comeback in law schools. See generally Steven I. Friedland, Towards the
Legitimacy of Oral Examinations in American Legal Education, 39 Syracuse
L. Rev. 627 (1988) (calling for the supplementary use of oral examinations
in law schools).
Since their earliest appearance, bar exams have been both praised and
condemned. For two excellent articles that, when read together, nicely list
the pros and cons of bar exams, compare Erwin N. Griswold, In Praiseof Bar
Examinations, 60 A.B.A. J. 81 (1974) (pros) with Leon Green, Why Bar
19961 ANECDOTAL HISTORY OF THE BAR EXAM

Examinations?, 33 Nw. U. L. Rev. 908 (1939) (cons). See also Richard L.


Abel, American Lawyers 40-73 (1989) (placing the bar exam into the larger
context of the American legal profession through the use of numerous
statistics and tables); Ken Myers, Bar Exams Under ExaminationAs Dean
Decries Wasted Time, Nat'l L.J., Oct. 17, 1994, at All (questioning the utility
of bar exams in light of the ABA's calls for increasing stress on skills
training).

7. (b) Although all states now have a permanent board of law examiners
whose operations are financed either partially or wholly through applicant
fees, New Hampshire was the first state to utilize such a system. Accord-
ingly, (a), (c) and (d) are wrong.
As has been noted elsewhere, New Hampshire's innovation was adopted
rapidly by other jurisdictions:
The first effort to improve the examining machinery took the form of
substituting for direct judicial examination a system of referring appli-
cants to uncompensated boards or standing committees....
[In time, however, it was recognized that such boards were unlikely to
function properly unless their] members [were] held to their duties by
appropriate financial arrangements.
This development first occurred in the small state of New Hampshire.
In 1872 the Superior Court, having finally regained from the legislature
power to enquire into the "suitable qualifications" of applicants, at-
tempted to revive the old county bar recommending system. Since this
proved ineffective under modern conditions, the court ruled in 1876 that
applicants were to be examined by itself or by a committee appointed by it.
In 1878 this committee was converted into a permanent board, and in
1880 it was allowed to finance itself out of applicants' fees....
By 1890 three other states - Ohio, Wisconsin and Connecticut - with
some variations of detail, had inaugurated similar systems. The Ohio
Supreme Court in 1882 followed the same course as the New Hampshire
tribunal by allowing compensation to its recently established centralized
examining committee.... By 1890, then, reasonably satisfactory models
for a permanent central examining board had been developed by four
jurisdictions out of forty-nine. New York's adoption of the idea in 1894
gave it a great impetus, and under the influence of the American Bar
Association it has now come to be regarded as an indispensable feature of
an orthodox bar admission system.
Alfred Z. Reed, Trainingfor the Public Profession of the Law 100-03 (1921)
(footnotes omitted). For a detailed look at how a modern state board
operates, see Thomas A. Pobjecky, The FloridaBoard of BarExaminers: The
ConstitutionalSafeguard Between Attorney Aspirants and the Public, 18 Nova
L. Rev. 1313 (1994).
376 GEORGETOWN JOURNAL OF LEGAL ETHICS [Vol. 9:359

8. (c) Birch found Lincoln to be a rather lenient bar examiner. As the


following account of his meeting with the future president makes clear, (a),
(b) and (d) are incorrect:
Probably typical of the examination imposed upon students is the one
described by Jonathan Birch of Bloomington, who had arranged with
Lincoln to examine him for admission to the bar. After adjournment of
the circuit court at Bloomington, Birch went to Lincoln's hotel room.
"I knocked at the door of his room, and was admitted," he related.
"How long have you been studying?" he asked. "Almost two years," was
my response. "By this time, it seems to me," he said laughingly, "you
ought to be able to determine whether you have the kind of stuff out of
which a good lawyer can be made." Then he asked in a desultory way the
definition of a contract, and two or three other fundamental questions, all
of which I answered readily, and I thought, correctly. Beyond these
meager inquiries, as I now recall the incident, he asked nothing more.
Meanwhile, sitting on the edge of the bed he began to entertain me with
recollections - many of them characteristically vivid and racy - of his
own practice and the various incidents and adventures that attended his
start in the profession. The whole proceeding was interesting yet so
unusual, if not grotesque, that I was at a loss to determine whether I was
really being examined or not. In due time we went downstairs and over to
the clerk's office in the courthouse, where he wrote a few lines on a sheet
of paper which he enclosed in an envelope and directed me to report with
it to Judge Stephen T. Logan [Lincoln's second partner who had earlier
been a Circuit Court judge], the other member of the examining commit-
tee at Springfield. The next day I went to Springfield where I delivered it
as directed. On reading it, Judge Logan smiled, and much to my surprise,
gave me the required certificate or license without asking a question
beyond my age, residence, and the correct way of spelling my name. The
note from Lincoln read: "My dear Judge: - The bearer of this is a young
man who thinks he can be a lawyer. Examine him if you want to. I have
done so and am satisfied. He's a good deal smarter than he looks to be.
Yours, Lincoln."
Len Y. Smith,Abraham Lincoln as a BarExaminer, B. Examiner, Aug. 1982,
at 35, 37.

9. (b) The other choices are fictional.


The details surrounding Long's admission to the bar have helped to
contribute to his larger-than-life persona:
Huey Long's oral bar examination in 1915 is part of Louisiana lore. When
asked by George Terriberry, an admiralty practitioner, what he knew
about admiralty, Long replied, "Nothing." When pressed about how he
would handle an admiralty matter, Long announced, "I'd associate Mr.
Terriberry with me and divide the fee with him." Long passed.
1996] ANECDOTAL HISTORY OF THE BAR EXAM

Robert Stevens, Law School: Legal Education in America from the 1850s to
the 1980s 32-33 n.43 (quoting T. Harry Williams, Huey Long 81 (1969)).

10. (b) Unlike the other jurisdictions, in which examinees are free to dress
as they wish, Kentucky and Virginia both require applicants to wear
business attire while taking the bar exam. Accordingly, (a), (c) and (d) are
wrong.
Virginia's requirement dates back to the 1960s. Telephone Interview with
Mary K. Altizer, Staff Administrator, Virginia Board of Bar Examiners
(Mar. 24, 1995). In 1993, Kentucky implemented a similar dress code. Since
that time, there has been a "marked difference in [the examinees'] attitude
and behavior. They are much more courteous, less belligerent, and more
cooperative." Telephone Interview with Patricia E. Gill, Executive Assis-
tant, Kentucky Board of Bar Examiners (Mar. 24, 1995).
For a general discussion of lawyer dress codes, see Charles W. Wolfram,
Modem Legal Ethics § 12.1, at 627 (1986) (noting judicial regulations on
attire of lawyers in courthouse), and Martin Fox, Bar Panel Tackles Sticky
Issue of Appropriate Garb for Women, N.Y. L.J., Dec. 23, 1991, at 1
(reporting on a New York ethics opinion that concluded that it was not
unprofessional for female attorneys to wear pants while appearing in court);
see also John Grisham, The PelicanBrief 12 (1992) (describing the heroine, a
Tulane law student named Darby Shaw, as "just one of the gang .. . [who]
adhered to the law school dress code of jeans and flannel shirts and old
sweaters and oversized khakis.").

11. (c) Indiana. From 1851 to 1933, Article 7, Section 21 of the Indiana
Constitution provided that, "Every person of good moral character, being a
voter, shall be entitled to admission to practice law in all courts of justice."
Bernard C. Gavit, Indiana'sConstitution and the Problem ofAdmission to the
Bar, 16 A.B.A. J. 595, 595 (1930).
Although almost every state had open bar admissions at one time or
another in the nineteenth century as part of a national movement to
demystify the law, only Indiana's experiment with lay lawyers lasted into the
twentieth century. Hence, (a), (b) and (d) are wrong. The provision finally
was repealed in 1932 by popular vote. In re Todd, 193 N.E. 865, 866 (Ind.
1935).
Ironically, Indiana today has one of the most restrictive bar admission
policies in the country due to its infamous "Rule 13." Adopted in 1973, the
rule requires every individual who wishes to take the Indiana bar exam to
provide proof that while in law school he or she completed at least 3 hours
of administrative law, 3 hours of business organizations, 4 hours of civil
procedure, 6 hours of commercial law (including contracts), 3 hours of
constitutional law, 4 hours of criminal law (including criminal procedure), 3
GEORGETOWN JOURNAL OF LEGAL ETHICS [Vol. 9:359

hours of evidence, 2 hours of legal ethics, 2 hours of legal research and


writing, 4 hours of property, 3 hours of taxation, and 4 hours of torts. Rules
for Admission to the Bar and the Discipline of Attorneys, Ind. Code Ann.
Court Rules Rule 13 (Burns 1995).
The American Bar Association has criticized Rule 13 (and South Caroli-
na's equally restrictive Rule 5A) on the grounds that it "interfere[s] with the
duty and responsibility of each individual law school faculty to determine
and periodically to revise the law school curriculum. . . [and] ... limit[s] the
opportunity for the law student to determine his or her own course of
study...." James P. White, Legal Education in the Era of Change: Law
School Autonomy, 1987 Duke L.J. 292, 297 (footnotes omitted). For a
further look at the pros and cons of the rule, see Robert M. Givan, Indiana's
Rule 13: It Doesn't Invite Conformity. It Compels Competency, Learning & L.,
Summer 1976, at 16, 16 (explaining the need for the rule), and Douglass G.
Boshkoff, Indiana'sRule 13: The Killy-Loo Bird of the Legal World, Learning
& L., Summer 1976, at 18, 18 (criticizing the rule).

12. (d) The Multistate Bar Examination (MBE) is a six-hour, 200-multiple


choice question exam prepared and scored by the National Conference of
Bar Examiners (NCBE). Arthur Karger, The Role of the NCBE in the Bar
Admission Process: Its FirstFifty Years, B. Examiner, Aug 1981, at 7, 10. First
given in February 1972, it is now in use in forty-six states, the District of
Columbia, Guam, Saipan and the U.S. Virgin Islands (the only jurisdictions
that do not use the MBE are Indiana, Iowa, Louisiana and Washington).
BARIBRI Digest 3 (1995). The test covers six subjects: Constitutional Law
(30 questions), Contracts (40 questions), Criminal Law (30 questions),
Evidence (30 questions), Real Property (30 questions) and Torts (40
questions). Id.
(a) and (b) are wrong because the MBE is only one part of the bar exam
in states that have chosen to adopt the MBE. Arthur Karger, The Role of the
NCBE in the Bar Admission Process:Its FirstFifty Years, B. Examiner, Aug.
1981, at 7, 11. (c) is wrong because the MBE has been adopted and because
it was the Eckler Committee, not Chief Justice Warren, who proposed the
MBE. Id.
The NCBE was founded in September 1931 to provide "a national
organization through which state boards of bar examiners could cooperate
with each other, with the law school community, and with the organized
bar." Id. at 7. The story behind the NCBE's decision to create the MBE is an
oft-told tale:
At the NCBE meeting in February, 1968, the subject under discussion
was means of implementing and improving the examining process. Robert
E. Seiler, then Chairman of the Board of Managers of the NCBE, and
1996] ANECDOTAL HISTORY OF THE BAR EXAM 379

more recently, Chief Justice of the Supreme Court of Missouri, an-


nounced that he proposed to appoint a special NCBE committee to make
an in-depth analysis of the bar examining process and that he was
designating [John Eckler, an Ohio attorney] to serve as the chairman of
the proposed committee.

The Committee held its first meeting, a brainstorming session, in


Chicago on January 24, 1969 to look at the entire bar examining process
and consider ways to improve it. The committee observed that all of the
state boards were doing about the same thing. Obviously, there was a
duplication of effort and inefficiency in the utilization of available skill and
talent. Following the first session, a summary and commentary on the
committee's discussion were distributed to the committee members as a
basis for future study.
In July, 1969, John Winterbottom of the Educational Testing Service,
developed a paper for the committee focusing on possible concerns. He
urged a careful study of the production and use of multiple choice
questions in bar examinations.
The second meeting of the committee was held in Dallas, Texas on
August 10, 1969 and considered the Winterbottom paper....
On the following day, at a meeting of the members of the NCBE, there
was a panel discussion by members of the committee on the subject of "A
Uniform Bar Examination - National or Regional: Is It Possible or
Practical?" ... During the discussion, Dean [Joe] Covington suggested
that since the title "National Bar Examination" raised the specter of loss
of local control of bar examinations, a better and more descriptive term
would be "Multistate Bar Examination." It was agreed that this denomi-
nation would underscore the intent of maintaining in each jurisdiction
local control of bar examinations and admission to the bar.

During its meetings, the committee concluded that a multistate exami-


nation would be favorably received by the various state boards since they
would retain control of their own examinations. The use of the proposed
examination would be entirely voluntary and it was expected that the
states would continue to offer their own essay examinations in conjunction
with the multiple choice test. There would be no interference with the
right and obligation of the states to determine pass-fail criteria.
The committee was satisfied that the multistate examination could be
expertly prepared with the assistance of a professional testing organiza-
tion to insure high standards of technical quality. A survey conducted by
the committee disclosed that there was considerable diversity in the
practices followed by the various states; that in most jurisdictions the bar
examination was a two-day test, but that in some it was a three-day test;
and that the subjects covered by the examination varied widely from
jurisdiction to jurisdiction. However, the committee found that all the
states covered the five basic subjects of contracts, torts, real property,
380 GEORGETOWN JOURNAL OF LEGAL ETHICS [Vol. 9:359

criminal law and evidence. The committee accordingly recommended that


the multistate examination be limited to those five subjects. (In recent
years, constitutio[]nal law has been added as a sixth subject.)
The committee further recommended that a one day multiple choice
examination be given twice a year, on the five subjects, and that each state
be permitted to use the second or third days for an additional section
employing essay questions. This section was to be prepared locally. Each
board would have an opportunity to compare the performance of its
applicants on each type of examination.

It was decided that the two multistate tests to be given each year would,
for reasons of security, have to be given on the same dates in all
[participating] states. Following a careful review of the dates on which bar
examinations were given in the various states, it was decided that the
multistate examinations should be held on the last Wednesday in Febru-
ary and the last Wednesday in July.
It was further decided that the examination would consist of 200
multiple choice questions, with 40 questions being allotted to each of the
five subject areas with which the examination was to deal. [When constitu-
tional law was later added as a sixth subject having 30 questions, the
number of criminal law, evidence and real property questions was reduced
from 40 to 30.] Each question was to be in the form of a statement of facts
followed by four suggested alternative answers - a best answer and three
distractors. It was felt that this examination would offer the most detailed
and thorough coverage ... ever available on any bar examination.
The locally prepared essay portion would be for one or more days of
examination, as the state boards desired, and would be administered
immediately before or after the Wednesday Multistate Bar Examina-
tion ....

Each state board retained the option to decide how it would use the
scores of its applicants on the Multistate Bar Examination.... However,
it was anticipated, that most states would require a pass score on a
combination of the two examinations.

For more than three years, the NCBE's initial "in-depth analysis"
committee and its bar examination committee worked diligently studying
the bar examination process and making recommendations based on its
study. The NCBE was certain that the Multistate Bar Examination would
not only improve the scope and quality of bar examinations, but would
solve the immense logistical problems faced by many examining boards,
including a general reduction in the time-lag between the giving of an
examination and the announcement of its results. The experience of the
Multistate Bar Examination program since the first examination on
February 23, 1972 has confirmed the Conference's optimism.
19961 ANECDOTAL HISTORY OF THE BAR EXAM

John Eckler, The Multistate Bar Examination: Its Origins and Objectives, B.
Examiner, Aug. 1981, at 15, 15-19. The history of the MBE also is told in
Daniel C. Blom, The Multistate Bar Examination:A New Approach, 44 B.
Examiner 8, 11-12 (1975).
Because the MBE is given on the last Wednesday of February and July,
many students are able to take two bar exams at the same time. Thus, for
example, an applicant in New York, where the local portion of the bar exam
is given on the Tuesday preceding the MBE, also can take the New Jersey
bar exam because its local portion is given on the Thursday following the
MBE. Similar combinations with other pairs of states also are possible,
although carefully coordinated travel plans and large amounts of stamina
often are needed (matching the Florida bar exam, a Tuesday-Wednesday
test, for example, with the Connecticut exam, a Wednesday-Thursday test,
requires the applicant to spend his or her first two days in Florida and then
fly to Connecticut on Wednesday evening). See Jeffrey Kanige, Nice Guys
Give Their Exam Last, N.J. L.J., Apr. 12, 1993, at 14 (describing the havoc
created for New Jersey and Connecticut when New York temporarily
moved its local portion to Thursday so as not to conflict with a religious
holiday).
Although the MBE is considered a success by bar examiners, many
outside the organization believe that, like other standardized tests, the
MBE places too much emphasis on memorization and not enough on skill
or aptitude:
There are untold law students, lawyers, and professors who decry the
MBE and MPRE and its multiple choice format. Offered answers may
include varying ranges of correct choices. Sometimes two answers are
correct and the test taker must choose between two correct answers - to
pick the best overall answer. Query: Is this a fair and intelligent indicator
of a person's knowledge of the law?...

...Consider some of these observations:


"The bar exam has very little relation to what you did in law school and
none at all to what you will do in practice - you might think of it as kind of
a rite of passage," ... As for Constitutional Law on the MBE, [Professor
Jeffries] concludes that "you can suffer not from knowing too little but,
from knowing too much because Multistate Constitutional Law does not
reward subtlety, sophistication, insight - these are not qualities valued on
the Multistate examination, instead, your examiners are looking for a kind
of mechanistic, buzzword oriented simplicity." University of Virginia Law
School Professor John C. Jeffries, Jr.
"What they test on this exam - pure and simple - is your ability to
remember.... You're going to get to the bar exam ...get to the first
question and you will know the answer.., then you will look down at your
four choices and the right answer won't be there. It is known as the best
GEORGETOWN JOURNAL OF LEGAL ETHics [Vol. 9:359

answer approach. You give ...the best wrong answer. The MBE is a very
bizarre test and it requires just the right touch ..... University of Virginia
Law School Professor Robert E. Scott.
BAR/BRI Digest, 2, 5 (1986) (emphasis in original). Are these tests
really benefitting either those who want to be lawyers or the public they
will serve if the tests are passed? What do they really test? Are the tests
turning out those who are adept at "mechanistic, buzzword oriented
simplicity"? And do they reject good legal minds who have depth and
insight?
These tests try to pigeonhole the law. Yes, put it in little egg-shaped
ovals (witness MBE and MPRE answer sheets). It does not sell with me.
The practice of law is heart and soul, learn through experience, study and
reason, test and weigh, absorb the facts, recognize legal principles,
conceptualize, balance, reason, formulate pleadings, write briefs, work
hard, be honest, pay your bills, give your word to a fellow attorney and
keep it, respect the courts, and above all, to help people. You cannot put
that in little egg-shaped ovals. Competence to practice law cannot be
measured by little ovals.
Mnemonics. Read that word. Mnemonics. Have you heard of it? It is a
technique for improving the memory by which a person links an easily
remembered word to legal concepts. Examples:
(1) Recalling Due Process requirements that must be complied with
before a valid judgment against a defendant can be entered is facilitated
by
M-Minimum contacts
O-Opportunity to be heard
P-Proper service of process
(2) You cannot generally commit a crime without your
M-Mens Rea
A-Actus Reus
(3) To remember the type of possession necessary to acquire real
property by adverse possession, think of possession that is
O-Open
C-Continuous
E-Exclusive
A-Actual
N-Notorious
(4) The contracts to which the Statute of Frauds applies (therefore
requiring a writing) may be identified by recalling
M-Marriage (contracts made upon consideration of marriage)
Y-Year (contracts incapable of being fully performed within one year)
L-Land (contracts for the sale of land)
E-Executor or Administrator (contracts to answer for a duty of a
decedent)
G-Goods (contracts for sale of some goods, e.g., UCC § 2-201(1)
requiresa writing when the price of goods sold exceeds $500)
1996] ANECDOTAL HISTORY OF THE BAR EXAM

S-Suretyship Agreements (contracts to pay the debt or default of


another)
Students today - studying for these MBE's and MPRE's - must
become buzzword artists. We should not substitute legal automatons for
persons who offer a point or series of points to advance a legal argument
which is questioned or disputed. Nor, for that matter, should we create
lawyers who are legal automatons rather than lawyers who can gather,
conclude, think, reason, reflect, and deduce. I visualize what would
happen if the older members of the Bar (to include myself), whether on
the plains of South Dakota or the industrial cities of the Eastern Sea-
board, had to employ mnemonics to pass an MBE and MPRE! A long
absence from an academic setting creates a difficult situation for an older
attorney to engage in mnemonics, choosing various answers which are
correct, and memorizing for multistate, multiple choice questions.
In re Voorhees, 403 N.W.2d 738, 740-42 (S.D. 1987) (Henderson, J., concur-
ring in part and dissenting in part) (footnotes omitted); see also Max A.
Pock, The Case Against the Objective Multistate BarExamination, 25 J. Legal
Educ. 66, 70 (1973) (arguing that the MBE does not test legal ability); David
M. White, The Definition of Legal Competence: Will the CircleBe Unbroken?,
18 Santa Clara L. Rev. 641, 671-74 (1978) (criticizing the MBE's effect of
restricting the number of applicants to the practice law).

13. (b) Bankruptcy. (a), (c) and (d) are wrong because the Multistate Essay
Examination (MEE), which consists of six thirty-minute essay questions,
covers the following subjects: business organizations (agency & partner-
ships and corporations), commercial transactions (commercial paper, sales
and secured transactions), conflict of law, family law, federal civil proce-
dure, and wills, estates and trusts (including future interests). BAR/BRI
Digest 4 (1995).
Like the Multistate Bar Examination, the MEE is the brainchild of the
National Conference of Bar Examiners. Id. It is designed to relieve indi-
vidual state boards of bar examiners from the onerous task of having to draft
bar exam essay questions, in much the same way that the MBE allows such
boards to avoid having to develop bar exam multiple choice questions. See
Ken Myers, Multistate Essay Examination Garners Some New Subscribers,
Nat'l L.J., Jan. 11, 1993, at 4 ("The MEE is ... an excellent vehicle for those
states that don't have the staff, funds or capacity to generate quality
essays. .. ."); (quoting John E. Holt-Harris, then chairman of the New
York State Board of Law Examiners); see also Marygold S. Melli, The
Multistate Essay Examination, B. Examiner, Nov. 1988, at 4, 6 (stating that
the overall goal was to produce a better exam than had been possible on an
individual state basis). Although the MEE has been available for adoption
GEORGETOWN JOURNAL OF LEGAL ETHICS [Vol. 9:359

since July 1988, to date only a few jurisdictions have shown any interest in
the MEE:
Currently the MEE is being administered in Arkansas, D.C., Idaho,
Illinois, Kansas, Maine, Mississippi, Missouri, Nebraska, Nevada, Utah
and West Virginia. Most jurisdictions employ it as half of the essay day,
but in Nebraska the MEE has been used exclusively - without any
questions drafted by the state examiners.
It is unclear whether or not the MEE will gain the wide acceptance that
its sister exams [the Multistate Bar Examination and the Multistate
Professional Responsibility Examination] now enjoy. Traditionally the
essays have kept each state's examination process unique, and it is
unlikely that [examinees] will face a standardized essay format anytime
soon.
BAR/BRI Digest 4 (1995). For a further discussion of the MEE, see
generally Marygold S. Melli, The Multistate Essay Examination, B. Exam-
iner, Nov. 1988, at 4 (describing the purpose, history, development and
grading of the 1988 MEE); Ken Myers, Multistate Essay Examination
Garners Some New Subscribers, Nat'l L.J., Jan. 11, 1993, at 4 (reporting that
the number of users of the MEE has tripled from 2,500 in 1992 to 7,500 in
1993); and Jane P. Smith, The New Multistate Essay Examination, B. Exam-
iner, Nov. 1992, at 13 (discussing changes in time limits, administrative
flexibility, test forms, subject matter and grading implemented in July 1993).
For an interesting article that describes how the New York State Board of
Law Examiners, which does not use the MEE, goes about drafting its essay
questions, see Matthew Goldstein, The Art and Science of Drafting the Bar
Exam, N.Y. L.J., July 26, 1994, at 1 (reporting that the questions are "the
handiwork of the five members of the Board of Law Examiners them-
selves").

14. (b) The Multistate Professional Responsibility Examination (MPRE) is


a two-hour, 50-question multiple choice test that is used in forty-two
jurisdictions. BAR/BRI Digest 3-4 (1995). It is designed to evaluate an
applicant's knowledge of the American Bar Association's (ABA) 1969
Model Code of ProfessionalResponsibility, 1983 Model Rules of Professional
Conduct and 1990 Model Code of JudicialConduct.Id. at 4.
The MPRE was developed by the National Conference of Bar Examiners
(NCBE) and first used in March 1980. Arthur Karger, The Role of the NCBE
in the BarAdmissionProcess: Its FirstFifty Years, B. Examiner, Aug. 1981, at
7, 11. Unlike the NCBE's other tests, applicants can take the MPRE while
still in law school in every jurisdiction - save one - that has adopted the
MPRE. BAR/BRI Digest 4. The lone exception is Florida, where candidates
must wait until after graduation. See In re Florida Board of Bar Examiners,
1996] ANECDOTAL HISTORY OF THE BAR EXAM

548 So. 2d 235, 236-37 (Fla. 1989) (rejecting a proposal to change the rule
out of a concern that "law students would devote less time than usual to
their law school course work throughout the weeks preceding the MPRE").
Because the MPRE's pass rate in many states in recent years has been in
the ninety percent range, a number of observers have questioned its utility.
In 1992, for example, Chief Justice Rosemary Barkett of the Florida
Supreme Court publicly ridiculed the MPRE and called for its abolition.
Mark D. Killian, BarExaminersto Ask Court to Replace MPRE, Fla. B. News,
Sept. 15, 1993, at 8. Ironically, when the Florida Board of Bar Examiners
subsequently sought permission to discard the MPRE, it was turned down.
See FloridaBoard of BarExaminers, 645 So. 2d 972, 975 (Fla. 1994) (finding
that the court was "not prepared to say that the examination serves no
purpose and should be eliminated"). It should be noted that Barkett had
left the court by the time the proposal was considered. See id. at 972
(noting that Barkett was not among the judges considering proposal).
(a) is wrong because the MPRE cannot determine whether a particular
applicant is trustworthy. (c) is wrong because the test is designed to be taken
at the time that admission to the bar is first sought. Despite this fact, many
jurisdictions regularly order disciplined lawyers to retake the MPRE as part
of their rehabilitation programs. See, e.g., In re Nassif, 504 N.W.2d 311, 315
(N.D. 1993) (ordering attorney to pass MPRE under North Dakota Stan-
dards For Imposing Sanctions). (d) is wrong because the test covers only the
disciplinary rules that have been promulgated by the ABA. BAR/BRI Digest
4 (1995).
For a further discussion of the MPRE, compare The MPRE Ten Year
Anniversary Issue, B. Examiner, May 1990, at 4 (reporting how acceptance
and performance of the MPRE has far exceeded expectations) with Edward
A. Adams, ProfessionalEthics Exam Session Is Marredby ChaoticProcedures,
Nat'l L.J., Dec. 28, 1987, at 4 (describing a particularly disastrous adminis-
tration of the MPRE at a site in Brooklyn, New York, in November 1987).

15. (d) The California performance exam is designed to go beyond the


traditional bar exam and test whether an applicant possesses such basic
lawyer skills as the ability to draft a complaint or write a letter to a client.
See Sec. of Leg. Educ. & Admissions to the Bar, Am. Bar Ass'n, Legal
Education and Professional Development - An Educational Continuum
280-81 (1992) (describing California's efforts to measure lawyering skills in
addition to those measured by traditional bar exams).
(a) is wrong because no test can predict whether a particular applicant
will commit malpractice. (b) is wrong because no state currently requires
lawyers to undergo periodic retesting. (c) is wrong because, although
community property questions often appear on the California bar exam, the
performance exam was not specifically designed to test the subject.
GEORGETOWN JOURNAL OF LEGAL ETHICS [Vol. 9:359

To date, observers remain divided whether performance tests reliably


measure lawyering skills or are merely much ado about nothing:
In July, 1982, Alaska became the first state to add a performance test to
its examination. The purpose of the test was to measure lawyering skills
that are not adequately tested by the other, traditional portions of the
exam ....

As originally devised, the test required applicants to write a memoran-


dum answering particular questions arising out of a given set of facts.
Applicants were provided a complete set of research materials, consisting
primarily of cases and statutes, some relevant and some irrelevant.
California has included a performance test on its bar examination since
1983. "The tests are designed to examine four broad categories of
lawyering competency: legal analysis, fact analysis, awareness of profes-
sional responsibility, and problem solving." The test does so by simulating
the types of documents with which lawyers work. Applicants are required
to complete a written assignment using facts contained in a file that
includes memos, letters, police reports, news articles, and the like, and
legal material consisting of statutes, cases and restatement sections. They
have three hours in which to answer two questions. Drafting assignments
have included outlining a deposition, drafting legislation, critiquing an
interview plan, writing briefs, writing a closing argument, drafting memo-
randa, writing a position paper, and writing investigation plans.
Colorado added a performance test to its examination in 1989 with the
goal of "test[ing] skills commonly used by lawyers practicing law compe-
tently." It consists of two thirty-minute questions which take the place of
three essay questions....
In the past two years, other states have begun considering whether to
follow the lead of Alaska, California, and Colorado ....
Proponents of adding a performance test to the traditional multiple-
choice and essay questions emphasize that the test successfully measures
attributes necessary for a candidate to provide competent legal assistance,
including attributes not measured by the traditional bar examination
questions....

A number of criticisms have been leveled, however, at the performance


tests that are currently in use .... [High on the list of complaints is the fact
that they test] the same lawyering skills of legal analysis and reasoning and
written communication that are measured by the traditional examina-
tions.
Id. at 280-82 (footnotes omitted). For a further discussion of performance
testing, see Alan Ogden, Performance Testing in Colorado, B. Examiner,
Nov. 1989, at 19 (discussing nature, history and results of Colorado's
performance testing), and Jane P. Smith, Performance Testing in California,
1996] ANECDOTAL HISTORY OF THE BAR EXAM

1983-1989, B. Examiner, Aug. 1989, at 17 (discussing goals, development,


administration and results of California's performance testing).

16. (d) It generally is agreed that there is no correlation between how a


person does on the bar exam and how he or she will fare in practice. See
Deborah L. Rhode, The Future of the Legal Profession: Institutionalizing
Ethics, 44 Case W. Res. L. Rev. 665, 690 (1994) ("No showing has ever been
made that performance either on bar exams or in law school correlates with
performance in practice."). As is often pointed out, Charles Evans Hughes,
who ran for President of the United States in 1916 and served as Chief
Justice of the U.S. Supreme Court from 1930 to 1941, failed the New York
State bar exam seven times. The Rodent, Society of Repeat BarExam Takers,
N.J. L.J., Sept. 26, 1994, at 20 ("The Rodent" is "an anonymous California
lawyer"). Accordingly, (a), (b) and (c) are wrong.
While running for Vice President in 1988, Dan Quayle responded to
charges that he was ill-prepared for high national office by reminding
audiences that Franklin D. Roosevelt had been a great president despite
failing the New York State bar exam numerous times. Much to Quayle's
embarrassment, research later revealed that Roosevelt had passed on his
first try. Bernard Weinraub, Campaign Trail: Now Quayle Fails a History
Exam, N.Y. Times, Sept. 29, 1988, at D29.
Although how one does on the bar exam is of no importance as long as
one passes, those who fail receive a terrible blow to their self-esteem and
often hide the results from their family and friends out of a sense of shame.
See, e.g., Neerings v. Utah State Bar, 817 P.2d 320, 320 (Utah 1991) (describ-
ing bar examinee's suit against state board of bar examiners for revealing his
failing score to his co-worker). In addition, many unsuccessful candidates
try to avoid telling their employers for fear that they will be fired. See Arleen
Stibelman, FailingBar Exam Less Risky at Big Finns, L.A. Daily J., Dec. 7,
1988, at 7 (noting that small California law firms usually are less patient with
those who fail than other types of California employers); Today's News
Update, N.Y. L.J., May 1, 1990, at 1 (reporting that after John F. Kennedy,
Jr., failed the New York State bar exam a second time, his employer, the
Manhattan District Attorney's Office, advised him that he would be fired if
he did not pass on his next try).
There is a vast literature on the subject of failure and the bar exam. See,
e.g., Monica Bay, Thick, You Fail: Facing the Bar a Second Time, Student
Law., Mar. 1983, at 20 (describing emotional reaction to failing California
bar exam); Stanley D. Chess, DearJohn, If Dan Quayle Could Pass, News-
week, May 14, 1990, at 57 (giving tips for passing the New York bar exam
after failing the first time); David Dumble, Your Worst Nightmare: You
Thought Taking the Bar Exam Was Bad. Confessions and Advice from One
Who Took It Again, Student Law., Mar. 1991, at 20 (giving advice to those
GEORGETOWN JOURNAL OF LEGAL ETHICS [Vol. 9:359

taking the bar exam a second time); Christi Harlan,AspiringLawyers Prepare


to Launch Appeal of Test Score: Group That Barely Failed Bar Exam in New
York Seeks to Overturn Results, Wall St. J., Dec. 12, 1988, at 4 (discussing
prospects of appealing one's failure of the New York bar exam); Leah
Rozen, Inside Room 400: Facing Up to Failingthe Bar, Am. Law., Mar. 1983,
at 43 (noting discouragement of those who fail the bar and their reasons for
retaking it); Thomas G. Ryan, Third BarExam Has Little Charm, Nat'l L.J.,
Feb. 2, 1987, at 13 (describing experience of retaking the Arizona bar
exam); Lynn Taylor, One Person in Four is Going to Flunk the Bar Exam,
Student Law., Mar. 1981, at 14 (describing those who failed the bar). See
also David Margolick, Year In and Year Out, He Took the Exam for 25 Years,
L.A. Daily J., Sept. 16, 1991, at 7 (profiling Maxcy Filer, who finally passed
the California bar exam after twenty-five years of trying).
Even those who pass the bar exam suffer considerable stress: first while
preparing for the test, then while taking it and finally while waiting for the
results. See Joan M. Cheever, A Scream and a Curse Open the Bar Exam,
Nat'l L.J., Aug. 7, 1989, at 8 (discussing applicants' anxiety before taking the
bar exam); Ron Coleman, The Endless Summer: A Would-Be Lawyer Endures
the Greenhouse - and the Madhouse - Effect in a Tough Training Season.
What More Can He Take? Only the Bar, Student Law., Mar. 1989, at 24
(describing applicant's growing terror while preparing for and taking the
bar exam); Jane Gross, BarExam: Ordeal and a Rite of Passage,N.Y. Times,
July 30, 1987, at B1 (comparing taking the bar exam as "a fraternity hazing"
and an "ordeal"); Stan Soocher, Do Some Questions Have 3 Right Answers?,
Nat'l L.J., Aug. 29, 1983, at 13 (describing fear during and after the bar
exam); Barbara K. Repa, The Waiting Game: You've Taken the Bar. Now,
How Do You While Away the Long Spell Between the Last Question and the
Final Word?, Student Law., Mar. 1986, at 40 (noting applicants' "frenetic
mood swings" while waiting for the results of the bar exam); E.R. Shipp,
After a Bar Exam, The Trial Begins, N.Y. Times, Aug. 5, 1981, at B1
(describing preparation for bar exam as "grueling," taking the exam as an
"initiation rite" and waiting for the results as "taxing").

17. (b) There is no evidence of a bar exam ever having been postponed or
cancelled due to UFOs.
(a) is wrong because in July 1985, Laura Beth Lamb, a lawyer with the
Securities and Exchange Commission, dressed up like her husband, Morgan
Lamb, and took the California bar exam for him. Wife Who Took Husband's
Exam Is Disbarred, Chi. Daily L. Bull., Aug. 8, 1989, at 1. Although
seven-months pregnant at the time and suffering from chronic diabetes, she
received the ninth-highest score on the exam. In re Lamb, 776 P.2d 765, 766
n.2 (Cal. 1989). Her deception was discovered in November 1985 after an
anonymous telephone caller tipped off the authorities. Id.
1996] ANECDOTAL HISTORY OF THE BAR EXAM 389

Upon being confronted with the evidence against her, Lamb confessed
and explained that she had been pressured into doing what she did by her
husband (who had failed the February 1985 California bar exam). See Wife
"ho Took Husband'sExam Is Disbarred,Chi. Daily L. Bull., Aug. 8, 1989, at
1 (reporting that Lamb pleaded no contest to the charges and said "she took
[the bar exam] out of fear [that if she didn't, her husband] would abuse
her"). Despite the fact that she divorced her husband and entered therapy,
the California Supreme Court took away her license. See In re Lamb, 776
P.2d 765, 768 (Cal. 1989) (holding that only the most "overwhelming
.evidence of mitigation" could prevent disbarment for something "so mor-
ally serious"); Wife Who Took Husband's Exam Is Disbarred,Chi. Daily L.
Bull., Aug. 8, 1989, at 1 (noting that Lamb was prohibited from reapplying
for admission for five years). For a similar case in which a husband was
disbarred for taking the Pennsylvania bar exam for his wife after she twice
failed to pass, see In re Pavilonis, 484 A.2d 1, 3 (N.J. 1984) (stating that the
"purpose of discipline is ...to protect the public from the attorney who
does not meet... [professional] standards ....).
For other instances of bar exam cheating, see In re Corrigan, 546 N.E.2d
1315, 1321 (Ohio 1989) (permanently banning a bar applicant who cheated
on the July 1988 Multistate Bar Exam from reapplying for admission to the
bar); In re Cummings, 9 So. 2d 614, 621 (La. 1942) (sanctioning attorneys for
trying to sell details of bar exam questions to examinees for $250); and State
ex rel. Turner v. Albin, 161 N.E. 792, 794 (Ohio 1928) (finding guilty of
contempt applicants who conspired with former bar employee to change
test scores surreptitiously). See also Kathleen M. May, A Human Behavior
Approach to Cheating on the Bar Exam, B. Examiner, Nov. 1994, at 18
(examining the nature and extent of cheating from law school to the
practice of law, processes underlying cheating and implications for reducing
cheating); James B. Tippin, Jr., Cheating,Evidence and Countermeasures,B.
Examiner, Feb. 1987, at 35 (explaining various conceptions of cheating,
methods of detection and remedies); Austin C. Wehrwein, Minnesota Justice
Quits Over Probe, Nat'l L.J., Mar. 25, 1985, at 10 (reporting on the resigna-
tion of Minnesota Supreme Court Justice John J. Todd for cheating on the
July 1983 Multistate Bar Examination after failing the July 1982 exam).
(c) is wrong because Kathleen E. McCasey publicly complained that an
essay question on the February 1994 California bar exam used sexual
language that was unnecessarily graphic:
...In a scenario - taken from a real case - about a landlord's agent
sexually harassing a tenant, the male agent says to the woman, "How
many times did you get laid last week?" and "How'd you like to have this
big sausage?" accompanied by a gesture to the man's groin.
Ms. McCasey, a graduate of the John F. Kennedy School of Law in
GEORGETOWN JOURNAL OF LEGAL ETHICS [Vol. 9:359

Walnut Creek, Calif., says that as test-takers reached that part of the test,
loud laughter was heard, disturbing other people's concentration.
"I felt like, 'How dare you do this when you didn't have to? Either you
are too dumb to not know you shouldn't do this, or you just don't care.'
Ms. McCasey said.
The "sausage" question was the talk of the post-mortem, she says.
Some people were offended, others thought it funny.
The next day, Ms. McCasey called the bar's office to express her
concerns and was told she had to write a letter. She did, but says she
believes that unless she files a formal complaint her concerns will be
ignored. She wants the results of that question to be thrown out from
consideration in scoring the California test unless it benefits the applicant.

Roger L. Meredith, chairman of the California bar committee, defends


both the process of selecting the questions and the question itself.
He says that particular question quoted a real sexual harassment case,
and using the specific language was crucial to the question because the
question asked applicants to analyze several allegedly harassing incidents.
He also says one of the drafters of the question was a woman, and more
than 50 people, including women, reviewed and pretested the question.
Ms. McCasey's complaint is the only one the bar has received out of
about 5,000 applicants, he says.
"The goal is to replicate problems lawyers are going to face in real life,"
Mr. Meredith says. "We don't seek to offend anyone, but these are
questions lawyers have to face."
Ken Myers, A Sexual HarassmentQuestion Spurs Complaint Over Bar Exam,
Nat'l L.J., Mar. 21, 1994, at A6; see also Judd F. Osten, BarExam Is No Place
for 'PoliticalCorrectness',Nat'l L. J., Apr. 18, 1994, at A16 (siding with the
bar examiners and stating that "it is readily apparent that it is Ms. McCasey
who 'just doesn't get it' ").
(d) is wrong because there have been several instances of bar exam
answers being inadvertently destroyed. See, e.g., In re July 1986 Ohio Bar
ExaminationApplicant No. 719, 574 N.E.2d 1047, 1049 (Ohio 1991) (refus-
ing to withdraw privilege to practice law when inadvertent destruction of
bar exams by unknown court personnel made it impossible to determine
examinees' results); John C. Metaxas, New York BarAnswers Lost; 542 Must
Retake Part of Test, Nat'l L.J., Aug. 26, 1985, at 4 (reporting on the
unexplained disappearance in July 1985 of hundreds of bar exams from the
New York State Board of Law Examiners' offices in Manhattan).

18. (c) D'Alemberte, a former dean of the Florida State University College
of Law, proposed that law students be allowed to take the bar exam after
their second year of law school. Ken Myers, ABA PresidentSuggests Students
Take Early BarExaminations, Nat'l L.J., Oct. 21, 1991, at 4. (a) and (b) are
1996] ANECDOTAL HISTORY OF THE BAR EXAM

wrong because D'Alemberte did not express any views on how bar exams
should be constructed. (d) is wrong for the same reason, although
D'Alemberte has championed women's issues throughout his career. This
fact caused him some grief when, shortly after he was tapped to head the
ABA, his daughter Gabrielle, a student at the University of Iowa, appeared
nude in a men's magazine. David Margolick, Playboy Appearance is Test of
Dad's Principles: ABA Chief's Daughter Bares It in Magazine, S.F. Chron.,
Sept. 9, 1991, at D3-D4.
D'Alemberte made his suggestion regarding the bar exam in October
1991 while delivering a speech at Stanford University to the Society of
American Law Teachers. During that talk he put forth the view that
students should spend the first two years of law school learning doctrine and
the third year engaged in clinical work and internships. He then suggested
that by allowing students to take the bar exam after their second year, the
transition from theory to practice would be accelerated. Ken Myers, ABA
President Suggests Students Take Early Bar Examinations, Nat'l L.J., Oct. 21,
1991, at 4.
Despite D'Alemberte's call for change, no state currently permits appli-
cants to take the general bar exam prior to their graduation from law school.
Although Georgia at one time allowed students to take its exam during their
last semester of law school, it ended this practice prior to the July 1995 exam
due to concerns that third-year students were spending too much time on
the bar exam and not enough time on their law school classes. Telephone
Interview with Joy C. Campbell, Assistant Director, Georgia Board of Bar
Examiners (Mar. 27, 1995).

19. (d) Bar applicants cannot be discriminated against on the basis of


where they have lived, currently live or plan to live in the future. Accord-
ingly, (a), (b) and (c) are wrong.
At one time, states often used geographical exclusions to limit bar
admissions. But in a series of cases decided in the 1980s, the U.S. Supreme
Court struck down all such exclusions. See Barnard v. Thorstenn, 489 U.S.
546, 553-54 (1989) (holding invalid the U.S. Virgin Islands' one-year
residency rule); Supreme Court of Virginia v. Friedman, 487 U.S. 59, 70
(1988) (finding that Virginia's admission rules must be applied in like
manner to residents and non-residents); Frazierv. Heebe, 482 U.S. 641, 646
(1987) (declaring invalid a federal court rule requiring bar applicants to live
in the state or maintain a local office); Supreme Court of New Hampshirev.
Piper,470 U.S. 274, 283 (1985) (deeming unconstitutional a New Hampshire
rule that limited bar admissions to state residents).
For a useful article on the subject, see Brian N. Corrigan & Eileen C.
Donovan, Note, Admission? Yes; Practice? No: New York's Inconsistent
Treatment of Nonresident Attorneys, 6 St. John's J. Legal Comment. 383
GEORGETOWN JOURNAL OF LEGAL ETHICS [Vol. 9:359

(1991) (examining the constitutionality of residency restrictions and examin-


ing less restrictive means for states to regulate the practice of law).

20. (b) In many states, newly admitted lawyers are required to attend
"Bridge-the-Gap" programs that are designed to help them make a smooth
transition from law school and the bar exam to the world of practice. Sec. of
Leg. Educ. & Admissions to the Bar, American Bar Ass'n, Legal Education
and ProfessionalDevelopment -An EducationalContinuum 289-90 (1992)
(footnote omitted) (reporting that approximately two-thirds of states offer
some kind of transition education). Accordingly, (a), (c) and (d) are wrong.
Although bridge-the-gap programs, also known as transition education
programs, have existed since at least the 1950s, it was not until 1980, when
New Hampshire made attendance at such programs mandatory for new
lawyers, that they began to assume their present form. Robert M. Jarvis,
Introduction, in Florida Legal Ethics § 1.8, at 1-1, 1-34 (Fla. B. ed., 1992).
Today, compulsory bridge-the-gap programs exist in almost all states, are
remarkably similar and suffer from the same sorts of problems:
... [M]any of the programs offered outside of law school aimed at new
practitioners are surprisingly uniform in program length, the topics cov-
ered, the class format, the identities of the speakers and the type of
program materials.
•.. The most common type of bridge-the-gap course offers a series of
lectures on substantive areas combined with detailed state-specific prac-
tice hints. Typical areas include bankruptcy, civil litigation, corporations
(small business), estate planning, wills and probate, criminal law, ethics,
family law, real property, and workers' compensation. Other topics often
found in bridge-the-gap programs include taxation and insurance law....

... The standard bridge-the-gap program lasts two or three, generally


consecutive, days. Most programs begin with a sign-in period and then a
welcoming speech. These are sometimes followed by an ethics or profes-
sionalism session. The day is most often split into at least two sessions,
with lectures covering different substantive topics.

•.. The overwhelming majority of the programs offer information in a


lecture format, which tends to be a passive method of learning requiring
little interaction or participation by the new lawyers....
Thus, the American programs of transition education often suffer from
a surfeit of substantive law and a concomitant lack of skills instruction....

... All of the programs•., rely principally on volunteer members of the


local bar to give the lectures in their programs. Generally, they are local
practitioners billed as experts in their respective fields. Sometimes they
are local judges .... Several of the programs have evaluation forms that
1996] ANECDOTAL HISTORY OF THE BAR EXAM

the participants fill out, one aspect of which deals with the speaker's
effectiveness and grasp of the information.
...Often, the speakers themselves develop the material that is distrib-
uted to the program's participants. The materials are generally directed at
those going into solo or small firm practice and cover a wide range of
topics typically encountered in a general practice. Many of the course
outlines are similarly of an exhaustive nature.

Sec. of Leg. Educ. & Admissions to the Bar, Am. Bar Ass'n, Legal Education
and ProfessionalDevelopment -An Educational Continuum 290-94 (1992)
(footnotes omitted).

21. (d) In nearly all states, once a person is admitted to the bar, he or she
may practice without any supervision. Accordingly, (a), (b) and (c) are
wrong.
At one time, however, many states required new lawyers to serve an
apprenticeship with an established lawyer:
The apprenticeship is a program for training new lawyers in a practice
setting. Although generally required in Commonwealth jurisdictions, the
apprenticeship has fallen into disfavor in the United States. At present,
only two states - Delaware and Vermont - continue to require law
school graduates to serve an apprenticeship. Both programs last approxi-
mately half a year and may be begun during law school. While completion
of the Delaware clerkship is a requirement for bar admission, the Ver-
mont internship may be completed within two years of admission.
The Delaware apprenticeship is considerably more structured than the
Vermont program. The Delaware clerk must complete 32 tasks which are
designed to develop the law clerk's lawyering skills in a "learning by
doing" atmosphere. For example, the clerk is required to attend various
courts and observe various proceedings in order to become familiar with
the Delaware judicial system and its procedures. The clerk must also
complete interactive tasks aimed at fostering lawyering skills. For ex-
ample, the clerk must help prepare a Superior Court motion, a Delaware
Supreme Court appeal, papers in connection with the commencement of
a lawsuit (including complaint, praecipe, and instructions to Sheriff), as
well as three memoranda of law and a will or trust instrument. Both the
law clerk and the preceptor are required to certify that the applicant
completed the clerkship.
In contrast, the Vermont Rules do not specify any tasks for the
neophyte lawyer, and they do not attempt to ensure that specific skills are
developed by outlining exercises to be completed, as does the Delaware
checklist. The Vermont intern, a graduate of an approved law school, is
simply required to "pursue[] the study of law within this state in the office
of a judge or a practicing attorney in this state" for the period of the
apprenticeship.
GEORGETOWN JOURNAL OF LEGAL ETHICS [Vol. 9:359

Within the past two decades, several other states, including New Jersey,
Pennsylvania, and Rhode Island, have replaced the requirement of an
apprenticeship with required attendance at a bridge-the-gap program....
The New Jersey program was discontinued for reasons which echoed
criticisms of "reading for the bar" of an earlier age. The experiences of
the New Jersey apprentices varied greatly, depending on the nature of the
law firm in which they served. The quality of the experience could not
easily be monitored .... [Slome clerkships [were] "a form of near-
peonage," and ... a six to nine month clerkship, with little or no salary,
could be a "serious economic burden" following three years of law school
for graduates already in debt and with families to support.
The apprenticeship systems currently retained in Vermont and Dela-
ware may be subject to similar criticisms.... [Moreover, nleither program
provides adequate assurance that the mentor is qualified to train the new
lawyer or will commit the time and effort to do so....
Sec. of Leg. Educ. & Admissions to the Bar, American Bar Ass'n, Legal
Education and Professional Development - An Educational Continuum
287-89 (1992) (footnotes omitted).

22. (d) In the United States, every lawyer is free to describe himself or
herself as an attorney, counselor or lawyer. Accordingly, (a), (b) and (c) are
wrong. Indeed, a lawyer may use any term that does not mislead the public.
See generally ABA Comm. on Professional Ethics and Grievances, Formal
Op. 81 (1932) (stating that "wherever a lawyer uses the term 'attorney' or
'counselor' as applied to himself," he should avoid misleading the public).
Originally, each designation had a precise meaning that, in addition to
reflecting the class structure and prejudices of English society, denoted
either a courtroom lawyer (advocate, barrister, counselor, narrator, ser-
jeant) or an office lawyer (attorney, conveyancer, pleader, solicitor). These
distinctions, which still exist in England, were dispensed with at an early age
in the United States. For a further discussion, see Theodore F.T. Plucknett,
A Concise History of the Common Law 215-30 (1956) (giving an historical
discussion of legal nomenclature); Max Radin, Handbook of Anglo-
American Legal History 248-60 (1936) (describing the history of the legal
profession from the thirteenth to the eighteenth centuries). See also William
Safire, On Language: We Wuz Robbed, N.Y. Times, Jan. 31, 1982, § 6, at 13
(discussing humorously the distinction between "attorney" and "lawyer").
The term "bar exam" owes its existence to the stratification of the English
legal profession. Whereas becoming a solicitor involves passing a qualifying
exam, becoming a barrister requires one to be "called to the bar." See
Maimon Schwartzschild, Class, National Character,and the Bar Reforms in
Britain: Will There Always Be An England?, 9 Conn. J. Int'l L. 185 (1994)
(comparing solicitors and barristers). Only those who have been so called
1996] ANECDOTAL HISTORY OF THE BAR EXAM

are permitted to pass through the railing that separates the back of the
courtroom (where the audience is seated) from the front (where the judge
and the lawyers are seated). Black's Law Dictionary 149 (6th ed. 1990).
Because the United States does not have a stratified bar, bar exams are used
to distinguish those who have been admitted to legal practice from those
who merely have a law degree:
Non-lawyers sometimes seem confused as to whether a certain person is
a lawyer. If someone finished law school but did not pass the bar exam, is
that person a lawyer? What if s/he passed the bar exam but never
practiced law? How about practicing law and then quitting - are you still
a lawyer?
The formal answer is that law school graduation, by itself, usually isn't
enough. You're not really a lawyer until you're admitted to practice law -
and that process usually requires a bar exam...
Raymond L. Woodcock, Take the Bar and Beat Me 117 (1991).
Regardless of how one describes himself or herself, all lawyers are
permitted to use the honorific "esquire" after their names. See Ask the
Globe, Boston Globe, Jan. 4, 1993, at 52 ("[T]he title belongs officially to
lawyers and other public officers."). In feudal times, the title referred to
those members of the English gentry who ranked just below knights. Id.
Although originally used only by men, the term now applies to both male
and female lawyers. Id.; Paula Bern, Male, Female Lawyers Deserve Same
Courtesy Title Despite Origin, Wash. Times, Mar. 9, 1992, at D3. See also
Antonelli v. Silvestri, 137 N.E.2d 146, 147-48 (Ohio Ct. App. 1955) (noting
that the term "esquire" has no actual connection to the legal profession).
That the United States continues to use so many different words to
describe members of the legal profession is due in large part to the fact that
lawyers fill so many different roles in American society:
A curious dualism afflicts the words used to describe members of the legal
profession. Lawyers, who are called that by nearly everyone who isn't one,
sometimes prefer to call themselves attorneys. Cribbing definitional lines
might be erected here, perhaps between lawyers as the generic term for
anyone graduated from law school and attorneys as those who actually
represent clients or, more narrowly, who represent clients in courts.
Attorney at common law described a person who served as a pleading
drafter, but now the usage applies to all lawyerly roles....
Charles W. Wolfram, Modern Legal Ethics § 1.1, at 4 (1986).

23. (a) A lawyer admitted in one state may ask a court in another state to
admit him or her in a given case. If the motion is granted, the lawyer is said
to be appearing pro hac vice (literally, "for this turn"). See Michael S.
Ariens, A Uniform Rule Governing the Admission and Practice of Attorneys
GEORGETOWN JOURNAL OF LEGAL ETHICS [Vol. 9:359

Before United States District Courts, 35 DePaul L. Rev. 649, 658 n.38 (1986)
(definingpro hac vice (citing Black's Law Dictionary1091 (5th ed. 1979))). In
Leis v. Flynt, a criminal case involving the owner of Hustler magazine, the
U.S. Supreme Court made it clear that being admitted pro hac vice is a
privilege, not a right, and the decision whether to grant such a request
resides in the sound discretion of the trial judge. Leis v. Flynt, 439 U.S. 438,
442 (1979).
Although most requests for admission pro hac vice are granted, over the
years several notable ones have been rejected. In In re Belli, for example, the
famous San Francisco trial lawyer Melvin Belli was denied his motion for
admission pro hac vice to retry a case because of his behavior during the first
trial. In re Belli, 371 F. Supp. 111, 112-13 (D.D.C. 1974). Similarly, in In re
Bailey, Boston criminal defense lawyer F. Lee Bailey was banned from
applying for admission pro hac vice in the New Jersey courts for one year
due to his unethical behavior in prior cases. In re Bailey, 273 A.2d 563, 566
(N.J. 1971).
For a further discussion of pro hac vice issues, see generally Timothy C.
Cashmore, Note, Due Process and Pro Hac Vice Appearances by Attorneys:
Does Any Protection Remain?, 29 Buffalo L. Rev. 133 (1980) (considering
attorney's interests in pro hac vice representation and protection of these
interests in light of broad judicial discretion), and Ballard J. Yelton, Note,
Analysis of Illinois' Restrictions on the Practiceof Law by Out-of-State Attor-
neys: Pro Hac Vice Model Rule Proposal, 16 Loy. U. Chi. L.J. 695 (1985)
(reviewing states' reasons for restricting the interstate practice of law and
discussing Illinois' rules on admission for out-of-state attorneys pro hac
vice).
(b) is wrong because the mere act of being admitted to practice in one
jurisdiction confers no automatic rights in any other jurisdiction.
(c) is wrong because each state has its own bar exam requirements. As
such, being admitted in one jurisdiction will not automatically qualify a
lawyer to sit for another state's bar exam. See Gail Fruchtman et al.,
Questions & Answers, 85 Law Libr. J.221, 224-29 (1993) (discussing mul-
tistate licensing of attorneys).
(d) is wrong because, although many states will allow a member of
another state's bar to join its bar without having to take its bar exam, this
right (known as admission on motion) does not exist throughout the
country:
Admission to practice law in one state does not give the lawyer a right to
practice in another state. A majority of the states, however, have extended
full membership to the state bar upon filing an affidavit that the individual
is admitted to practice in another state and by fulfilling certain special
conditions. This method for admission often waives the examination
1996] ANECDOTAL HISTORY OF THE BAR EXAM

requirement for bar membership, but not other qualifications such as


character review. Some states require that the applicant have attained a
particular score on the multistate portion of the examination or that the
applicant simply take the state law portion of the examination to qualify
for admission on motion. Other requirements, such as a minimum number
of years of practice in the home state, are imposed to prevent applicants
from flocking to the states with liberal admission criteria. Once the motion
or application is filed, the state bar admits the lawyer to practice.
Admission by reciprocity is very common in all but the popular and
populous sun-belt states [such as Arizona, California, and Florida] that
wish to protect their members from retiring easterners.
John F. Sutton, Jr. & John S. Dzienkowski, Cases and Materials on the
ProfessionalResponsibilityof Lawyers 78 (1989).
Because of the frustration experienced by the increasing number of
lawyers whose practices cut across state lines, there have been repeated
calls in recent years for the creation of a national bar exam that would grant
successful applicants admission in all fifty states and the District of Colum-
bia. To date, however, no serious efforts have been undertaken to create
such an exam. For a sampling of the arguments for and against universal
admission, see Samuel J. Brakel & Wallace D. Loh, Regulating the Interstate
Practice of Law, 50 Wash. L. Rev. 699, 713-14 (1975) (calling for a "na-
tional" bar exam supplemented by a local exam); Marvin Cominsky & Philip
C. Patterson, The Case for a Federally Created National Bar by Rule or by
Legislation, 55 Temp. L.Q. 945 (1982) (urging federal legislation permitting
the multi-state practice of law); Michael K. McChrystal, Legitimizing Reali-
ties: State-Based BarAdmission, National Standards,and Multistate Practice,
3 Geo. J. Legal Ethics 533 (1990) (arguing for state-based bar admission).

24. (c) Admission to the bar of the U.S. Supreme Court will be granted to
any lawyer who has been admitted to the highest court of his or her state or
territory for at least three years, is in good standing, and pays the necessary
fee (currently $100). See Michael S. Ariens, A Uniform Rule Governing the
Admission and Practice of Attorneys Before United States District Courts, 35
DePaul L. Rev. 649, 651 n.14 (1986) (quoting rule concerning eligibility to
admission to practice before Supreme Court).
(a) is wrong because a justice's nomination is not necessary (sponsorship
by two members of the Supreme Court bar, however, is required). Sup. Ct.
R. 5, 115 S. Ct. CCCXI, CCCXIV (1995) (giving the requirements for
admission to the Supreme Court bar). (b) is wrong because no test is
required to join the Supreme Court bar. (d) is wrong because the Supreme
Court does not require any particular level of skill or experience prior to
admission.
From time to time, some observers have complained about the Supreme
GEORGETOWN JOURNAL OF LEGAL ETHICS [Vol. 9:359

Court's "rubber-stamping" of admission applications. See, e.g., In re Rose, 71


L. Ed. 2d 862 (1982) (Burger, C.J., dissenting) ("[T]he applicant seeks
admission merely to receive the 'stamp of approval' from this Court. . . , a
practice... that has become all too frequent in recent years." (citing In re
Caplinger,101 S. Ct. 2303, 2306 (1981)); Charles W. Wolfram, Modem Legal
Ethics § 15.2.4, at 853 (1986) ("[T]housands of lawyers, who will never file
another motion there, [apply for admission to obtain] an impressive certifi-
cate for their office walls.").
Although the Supreme Court has not felt any need to tighten its admis-
sion standards, in recent years an increasing number of lower federal courts
have made it tougher for lawyers to join their bars and a few, such as the
Southern District of Florida, have gone so far as to institute their own
written bar exams. See generally Michael S. Ariens, A Uniform Rule Govern-
ing the Admission and Practice of Attorneys Before United States District
Courts, 35 DePaul L. Rev. 649 (1986) (offering an exhaustive study of
district court admission practices). For a further discussion, see Brown v.
McGarr,774 F.2d 777, 778-79 (7th Cir. 1985) (upholding a Northern District
of Illinois rule that requires lawyers to have four "qualifying units" of trial
experience before being admitted to the district's trial bar); Marvin E.
Frankel, CuringLawyers' Incompetence: Primum Non Nocere, 10 Creighton
L. Rev. 613, 626 n.50 (1977) (noting that the Southern District of Ohio has
required applicants to pass a written bar exam for more than fifty years);
Malcolm R. Wilkey, A Bar Examination for Federal Courts, 61 A.B.A. J.
1091, 1092 (1975) (proposing the creation of a "United States Bar").

25. (b) An otherwise-qualified foreign lawyer will be accorded the same


admission rights as a U.S. citizen. See, e.g., Bashirv. Supreme Court of Ohio,
652 F.2d 641, 642-43 (6th Cir. 1981) (holding that a Pakistani lawyer who
was not admitted in any American jurisdiction could be denied automatic
admission to the bar but not the right to take the bar exam); Teare v.
Committee on Admissions, 566 A.2d 23, 29 (D.C. 1989) (holding that a
foreign lawyer who did not possess a law degree from or attend an ABA
approved law school could not be kept out of the bar but could be made to
take the bar exam).
(a) is wrong because no treaty currently exists regulating the transna-
tional movement of lawyers. See Jonathan Barsade, The Effect of EC
Regulations Upon the Ability of U.S. Lawyers to Establish A Pan-European
Practice,28 Int'l Law. 313, 328-29 (1994) (proposing a treaty solution). As
such, every country is free to set its own rules regarding the admission of
foreign lawyers. Needless to say, this has resulted in a bewildering number
of rules and regulations, many of which are extremely hostile to foreign
lawyers. See Richard L. Abel, TransnationalLaw Practice,44 Case W. Res.
L. Rev. 737, 750-80 (1994) (describing "forms of regulation that have sought
1996] ANECDOTAL HISTORY OF THE BAR EXAM

to obstruct or control [the] growth [of transnational practice]"); Jonathan


Barsade, The Effect of EC Regulations Upon the Ability of U.S. Lawyers to
EstablishA Pan-EuropeanPractice,28 Int'l Law. 313, 314-26 (1994) (explain-
ing the EC's rules harmonizing legal standards on the movement of attor-
neys among member states); Roger J. Goebel, ProfessionalQualificationand
EducationalRequirementsfor Law Practicein a Foreign Country: Bridging the
CulturalGap, 63 Tul. L. Rev. 443, 462-523 (1989) (discussing preparation
for transnational legal practice and the extent of the right to engage in such
practice); Kelly C. Crabb, Note, Providing Legal Services in Foreign Coun-
tries: Making Room for the American Attorney, 83 Colum. L. Rev. 1767,
1768-88 (1983) (evaluating other nations' concerns about American legal
services abroad and suggesting regulatory guidelines to meet those fears).
In an attempt to make it easier for their own lawyers to practice in foreign
countries, a number of U.S. jurisdictions, including California, Florida,
Hawaii, New York and Texas, now permit foreign country lawyers to apply
for a limited license without having to take the bar exam. Such a license
allows the holder, who is designated a "foreign legal consultant," to advise
U.S. and foreign clients on the law of the licensee's home country. For a
useful overview of such programs, see Julie P. Fallon, Note, Florida's
ForeignLegal Consultancy Rule: A Potential Threat to the Public, 7 Fla. J. Int'l
L. 149, 150-52 (1992) (describing the history of foreign legal consultancy
rules).
(c) is wrong because an applicant will not be denied admission solely on
the basis of having received his or her primary instruction in a legal system
other than the common law. It is true, however, that the credentials of such
applicants often are screened carefully by the bar examiners. See, e.g., Sodha
v. New York State Bd. of Law Examiners, 431 N.Y.S.2d 885, 887 (Sup. Ct.
1980) (affirming bar examiners' decision to deny Indian petitioner's applica-
tion for substantial educational equivalence); cf. Anthony E. Davis, The
Evaluationof ForeignAcademic and ProfessionalLegal Credentials,B. Exam-
iner, May 1987, at 28, 34 (arguing for more careful screening of foreign
credentials by bar examiners); The Comm. on Legal Educ. & Admission to
the Bar, Revised Report on The Rules ForAdmission As Attorneys ForForeign
Educated Lawyers, 43 Rec. Ass'n B. City N.Y. 711 (1988) (proposing to the
New York bar examiners the adoption of more workable rules for evaluat-
ing foreign credentials).
(d) is wrong because lack of U.S. citizenship will not prevent an otherwise-
qualified applicant from being admitted to the bar. See In re Griffiths, 413
U.S. 717, 729 (1973) (holding that the Fourteenth Amendment guarantees
the right not to be discriminated against on the basis of citizenship when
applying for admission to the bar); Dingemans v. Board of Bar Examiners,
568 A.2d 354, 357 (Vt. 1989) (holding that the state's refusal to allow a
GEORGETOWN JOURNAL OF LEGAL ETHICS [Vol. 9:359

Dutch national to take its bar exam impermissibly burdened the federal
immigration system).

IV. ANSWERS TO ESSAY QUESTIONS


A.
Unfortunately, Harris is unlikely to be successful in his quest.
The requirement that an applicant be a graduate of an accredited law
school has been consistently upheld. See, e.g., LaNave v. Minnesota Supreme
Court, 915 F.2d 386, 388 (8th Cir. 1990) (upholding Minnesota's rule
requiring graduation from an accredited law school for admission to the bar
by exam), cert. denied, 500 U.S. 923 (1991); In re Oliver, 413 S.E.2d 435, 437
(Ga. 1992) (affirming graduation from an approved law school as a prereq-
uisite for admission to the Georgia bar); Cline v. Supreme Court of Georgia,
781 F.2d 1541, 1543 (11th Cir. 1986) (upholding Georgia rule requiring that
applicant graduate from a law school approved by either the ABA, the
American Association of Law Schools, or the Georgia Board of Examiners
to take state bar exam); In re Murphy, 393 A.2d 369, 374 (Pa. 1978) (per
curiam) (approving Pennsylvania "comity rule" on admission to the bar
exam and admission to the bar "on motion", premised upon applicant
possessing a law degree from a law school accredited by the ABA), cert.
denied, 440 U.S. 901 (1979); Moore v. Supreme Court of South Carolina, 447
F. Supp. 527, 529 (D.S.C. 1977) (upholding South Carolina rule providing
that no person shall be admitted to the practice of law unless he or she is a
graduate of an ABA approved law school), aff'd mem., 577 F.2d 735 (4th
Cir. 1978), cert. denied, 439 U.S. 984 (1978); Lombardi v. Tauro, 470 F.2d
798, 800 (1st Cir. 1972) (upholding court rule requiring candidates to be
graduates of qualified law schools for admission to the Massachusetts bar),
cert. denied, 412 U.S. 919 (1973).
. The reason for such a rule has been stated many times: it ensures that
those granted a license to practice law have the educational training
necessary to do so competently. Although at one time each state undertook
the task of certifying law schools, only a few (most notably California) still
do so. Geoffrey C. Hazard et al., The Law and Ethics of Lawyering 861 (2d
ed. 1994). Instead, since 1928 most states have relied on the ABA to do the
job for them:
The petitioner contends that the board did not articulate a valid reason
for denying his application to sit for the bar examination and therefore he
has been denied his constitutional right to practice law. The board denied
the petitioner's application because he had not graduated from an ABA-
approved law school. The board recognized that it was dealing with an
eligibility requirement which it did not possess discretion to modify or
1996] ANECDOTAL HISTORY OF THE BAR EXAM

waive. The petitioner's argument boils down to a constitutional challenge


to the facial validity of S.J.C. Rule 3:01, § 3.3 [which requires graduation
from an ABA-approved law school].
This court has previously observed that the ABA approval requirement,
as well as a board of bar examiners' or similar authority's refusal to waive
that requirement, has been upheld by numerous State and Federal courts
in the face of constitutional challenges.... A State's substantial interest
in the qualifications of those it admits to the practice of law within its
boundaries provides the basis for broad power to establish standards for
licensing legal practitioners.... Such standards will be upheld under the
due process and equal protection clauses of the Fourteenth Amendment
to the United States Constitution so long as they have a rational connec-
tion with an applicant's fitness or capacity to practice law....
There is clearly a direct rational connection between the requirement
of graduation from an accredited law school and an applicant's fitness to
practice law. The ABA standards relating to the accreditation of law
schools provide assurance that applicants to the bar "have experienced a
generally uniform level of appropriate legal education."... The Supreme
Court of Florida explained its rationale for relying on ABA accreditation
in LaBossiere v. Florida Bd. of Bar Examiners, 279 So. 2d 288, 289 (Fla.
1973): "We were persuaded to follow the American Bar Association
standard relating to accreditation of law schools because we sought to
provide an objective method of determining the quality of the educational
environment of prospective attorneys. This was deemed especially neces-
sary because of the rapid growth in the number of educational institutions
awarding law degrees. We wished to be certain that each of these many
law schools provided applicants with a quality legal education, but we
were unequipped to make such a determination ourselves because of
financial limitations and the press of judicial business." ...
In re Tocci, 600 N.E.2d 577, 581 (Mass. 1992) (footnotes and citations
omitted).
In order to be (and remain) accredited by the ABA, a law school must
meet certain minimum objective as well as subjective standards:
... [Tihe ABA, acting through its Section of Legal Education and
Admissions to the Bar, has promulgated standards for "approved law
schools" and applies these standards through extensive scrutiny of new
schools and sabbatical inspections (once every seven years) of previously
approved law schools. The ABA is recognized by the U.S. Department of
Education as the only accreditation agency for degrees in law.

Accreditation by the ABA requires maintenance of specified standards


as to curriculum, law library, classroom facilities and faculty. Compliance
is enforced by periodic inspections and by the sanction of withdrawal of
accreditation. The standards permit a substantial range in faculty-student
ratio (but no more than 30 students per full-time faculty member),
GEORGETOWN JOURNAL OF LEGAL ETHICS [Vol. 9:359

pedagogical procedure (e.g., "lecture" versus "Socratic method"), clinical


instruction, training in legal writing and research, subject matter of
courses and relative balance between theory and "nuts and bolts."
According to the rules in every state, accreditation of a law school by
the ABA constitutes accreditation for purposes of fulfilling that state's
requirement of graduation from an approved law school. A few states
such as California separately accredit law schools, so that a law school may
be accredited for purposes of admission by a state such as California but
not for purposes of admission in other states....
Geoffrey C. Hazard et al., The Law and Ethics of Lawyering 860-61 (2d ed.
1994). For a further discussion of the ABA's role in accrediting law schools,
see Robert MacCrate, Legal Education: Past Developments, Present Sta-
tus, and Future Possibilities, Address at Wake Forest University School of
Law (Mar. 27, 1995), in 30 Wake Forest L. Rev. 261 (1995) (discussing the
ABA's efforts to strengthen the education and training of lawyers to
improve the legal profession), and Susan K. Boyd, The ABA's First Section:
Assuring a Qualified Bar (Section of Legal Educ. & Admissions to the Bar,
American Bar Ass'n ed., 1993) (describing the first 100 years of the ABA's
accreditation committee). See also Roger C. Crampton, The Regulation of
Legal Education, 32 J. Legal Educ. 159 (1982) (discussing the ABA's
increased regulation of legal education and resistance to these develop-
ments).
Despite its long track record, since the early 1980s a growing number of
observers, both inside and outside legal education, have questioned whether
the ABA should continue to serve as the national accrediting agency for law
schools. In In re Laclede School of Law, for example, the Missouri Supreme
Court was asked to recognize a new law school that did not meet the ABA's
standards. In re Laclede School of Law, 700 S.W.2d 81, 82 (Mo. 1985).
Because the school was the only one in the state with a part-time program,
its supporters argued that an exception was in order. They also criticized the
ABA's standards as being overly rigid and insensitive to local concerns. See
id. (noting the unaccredited law school's arguments that "those who must
support themselves by working should be able to ... be admitted into the
legal profession" and that "full compliance with [ABA] standards is not
essential to an adequate education"). The court, however, rejected the
application. Id. at 84.
Even established law schools, who benefit from the ABA's standards
(because they make it harder for new schools to open), increasingly have
complained that the standards put them to unnecessary expense, are
applied in an arbitrary fashion and do not ensure quality education. In
November 1993, the Massachusetts School of Law (MSL) in Andover, after
failing to get ABA accreditation, did more than just complain: it sued the
ABA in federal court and claimed that the ABA's standards exist princi-
1996] ANECDOTAL HISTORY OF THE BAR EXAM 403

pally to enhance the salaries and working conditions of law professors.


Massachusetts Sch. of Law, Inc. v. American Bar Ass'n, 872 F. Supp. 1346,
1379 (E.D. Pa. 1994). See also Ken Myers, Do Law School GradersMeasure
Up? Top-Tier Deans and a Rebel's Lawsuit Shake up the Debate over ABA
Accreditation, Nat'l L.J., Feb. 20, 1995, at Al (reporting allegations that the
"ABA's monopoly of accreditation violates antitrust law").
In June 1995, the long years of protest against the ABA suddenly bore
fruit: on a single day the federal government both filed a civil antitrust suit
against the ABA and announced that it had entered into a consent decree
with the organization. Ken Myers, Settlement Will Mean Change in ABA's
Accreditation Process, Nat'l L.J., July 10, 1995, at A15. Under the agree-
ment, the ABA must substantially revise its accreditation practices (particu-
larly with respect to faculty salaries, student-to-faculty ratios, teaching
loads, sabbaticals and bar exam preparation courses) so as to make compli-
ance less costly. Id. The net result is expected to be that law schools will
spend more money on student needs and less on faculty salaries and fringe
benefits. For a further discussion, see Richard C Reuben, Accreditation on
Review, A.B.A. J., Oct. 1995, at 107 (presenting reactions to the ABA
consent decree with the U.S. Department of Justice).
Before leaving the accreditation issue, one other point must be noted. It
is possible that Harris' law school will one day receive accreditation. Even if
it does, however, such future accreditation will be of absolutely no help to
Harris because students who graduate before accreditation is granted are
viewed as having attended an unaccredited school. See, e.g., In re Klein, 259
So. 2d 144, 145 (Fla. 1972) (per curiam) ("We agree with the Board's
rejection of petitioner's application as it now stands. His graduation pre-
ceded accreditation [of John Marshall Law School] by some 27 months.");
In re Batten, 428 P.2d 195, 195 (Nev. 1967) (denying a petition for admission
to the bar to a graduate of Fordham University, whose graduation preceded
accreditation). As a result, new law schools seek to receive provisional
accreditation before their first class graduates; those that are unsuccessful
can do nothing but watch as their charter classes are permanently frozen out
of the bar exam. See Plechnerv. Widener College, Inc., 418 F. Supp. 1282,
1301 (E.D. Pa. 1976), aftd, 569 F.2d 1250 (3d Cir. 1977) (rejecting plaintiffs'
contention that even without Delaware Law School's affiliation with Widener
University, accreditation would still have followed prior to graduation of the
law school's first class); Ken Myers, Students' Suit: Delay Graduation Until
After ABA Accreditation, Nat'l L.J., Nov. 7, 1094, at A17 (reporting on a
lawsuit brought by fifty-eight Texas Wesleyan law students who claim that
they were forced to graduate early).
Although Harris' challenge of the accreditation rule appears to be
doomed, he may be able to prevail on his request for a waiver from the
GEORGETOWN JOURNAL OF LEGAL ETHICS (Vol. 9:359

college degree requirement. Recently, many states have become much less
demanding with respect to pre-law college study:
Upon consideration, the Court has concluded to dispense with the strict
requirement of an undergraduate degree. We note that the majority of
other states do not have such a requirement, and we conclude that
disputes over credentials evaluations are expensive, time-consuming, and
unnecessary. The Court has determined that the sole educational require-
ment of an applicant should be graduation with a J.D. or LL.B. degree
from a law school approved by the American Bar Association. The vast
majority of such graduates will have obtained satisfactory undergraduate
degrees, and we are confident that approved law schools will only permit
the admission of those without such undergraduate degrees in worthy
cases.
Florida Bd. of Bar Examiners re Amendment to Rules of the Supreme Court
Relating to Admissions to the Bar, 603 So. 2d 1160, 1160 (Fla. 1992) (per
curiam). Accordingly, if Harris is ever able to overcome the fact that his law
degree is not from an accredited law school, it may be possible for him to
convince the court to allow him to take the bar exam even though he does
not possess a college degree.

B.
With the exception of her demand to see her test papers, all of Smith's
claims are likely to be rejected.

1. Limit on the Number of Times an Applicant Can Take the Bar Exam
Many states limit the number of times an applicant can take the bar exam.
See Stephen Gillers, Regulation of Lawyers: Problems of Laws and Ethics 624
(4th ed. 1995) ("Twenty-four American jurisdictions limit the number of
times an applicant may take the bar examination. Puerto Rico's is six times.
Others range from five times ... down to two times.... "). Although such
rules have been challenged on a number of grounds, they have been upheld
as necessary to protect the integrity of both the bar exam and the legal
profession. See, e.g., Jones v. Board of Comm'rs, 737 F.2d 996, 1002 (11th Cir.
1984) (upholding Alabama's five-time limit); Poats v. Givan, 651 F.2d 495,
499-50 (7th Cir. 1981) (upholding Indiana's four-time limit).
Smith's best strategy is to ask for a waiver of the rule in the event that she
fails her third try. In order to obtain such relief, she most likely will have to
demonstrate that there is a compelling reason to believe that she will pass if
given another chance. See, e.g., In re Thao, 635 A.2d 1195, 1196 (R.I. 1994)
(holding that an applicant, whose native language was not English and who
had failed the bar exam three times, would be allowed to take the test a
1996] ANECDOTAL HISTORY OF THE BAR EXAM

fourth time if she successfully completed a course in English writing,


composition and reading); In re Fischer, 425 A.2d 601, 604 (Del. 1980)
(holding that applicant who failed the multistate portion of the bar exam
three times, but who passed all other portions, would be permitted to take
exam again if he completed a bar exam review course).

2. Claim that Questions and Answers Must Be Made Available for Review
Generally speaking, bar examiners are permitted to withhold such infor-
mation as is necessary to permit them to reuse their exams. See National
Conference of Bar Examiners v. Multistate Legal Studies, Inc., 692 F.2d 478,
483 (7th Cir. 1982) (holding that multistate bar exam could be copyrighted
without disclosure of its contents), cert. denied, 464 U.S. 814 (1983). By the
same token, however, it has been recognized that unsuccessful bar appli-
cants are entitled to some consideration.
In an attempt to balance these competing concerns, courts have held that
applicants who fail must either be given a right to inspect their papers (at
such times and on such conditions as the examiners may specify) or the right
to take the bar exam as many times as they wish. See, e.g., Fields v. Kelly, 986
F.2d 225, 228 (8th Cir. 1993) (holding that bar applicant's right to inspect
public records was not infringed when he was given limited opportunity to
review the exam); Bowles v. Askew, 448 S.E.2d 191, 192 (Ga. 1994) (holding
that denial of failed bar applicant's request to review exam was not a denial
of due process given that there was no limit to number of times applicant
could take the bar), cert. denied, 115 S. Ct. 1111 (1995).
Because Smith's state currently limits the number of times that she can
take the exam, chances are good that she will be granted some access to her
test papers. How much access she will be given, however, will depend on
what the examiners consider reasonable for her to learn from her past
mistakes. For a further discussion, see generally David P. Chapus, Annota-
tion, FailedApplicant's Right of Access to Bar Examination Questions and
Answers, 57 A.L.R.4th 1212 (1987) (examining cases that have decided
whether, and to what extent, individuals who have failed the bar exam can
inspect questions and answers from the exam).

3. Claim that Bar Exam is Culturally Biased


The assertion that the bar exam is culturally biased against minorities is
an old one. Unfortunately for Smith, courts routinely reject such claims
absent a showing of an intent to discriminate. See, e.g., Richardson v.
McFadden, 563 F.2d 1130, 1131 (4th Cir. 1977) (en banc) (stressing that
erroneous result in individual applicant's case does not amount to denial of
due process or equal protection), cert. denied, 435 U.S. 968 (1978); Tyler v.
GEORGETOWN JOURNAL OF LEGAL ETHICS [Vol. 9:359

Vickery, 517 F.2d 1089, 1100-01 (5th Cir. 1975) (holding that where bar
examiners have demonstrated absence of intentional racial discrimination,
disproportionate failure of black applicants does not trigger strict scrutiny),
cert. denied, 426 U.S. 940 (1976); Pettitv. Ginerich, 427 F. Supp. 282, 293 (D.
Md. 1977) (holding that facially neutral bar exam, which serves legitimate
state purposes, is not violation of Equal Protection Clause solely because
black applicants are disproportionately impacted), affdper curiam, 582 F.2d
869 (4th Cir. 1978).
Minority candidates, however, tend to do much worse on bar exams than
other candidates, even given special help. In New York City, a program
known as PALS, or Practicing Attorneys for Law Students, works with
minority applicants in a supportive, one-on-one setting. See Jennifer Kleiner,
Aid for Minority Students with Bar Exam, N.Y. L.J., July 27, 1995, at 2
(describing the PALS program and goals). Despite its efforts, PALS has had
only marginal success in raising the pass rate of minority students. See id.
(pointing out that on July 1992 bar exam, the pass rate for white students
was eighty-two percent, for PALS students forty-eight percent and for
minority students overall thirty-seven percent).
Although a number of theories have been advanced to explain this
phenomenon, none are particularly satisfactory. Likewise, a solution to the
problem does not appear to be close at hand. For a further discussion, see
generally Symposium, NationalConference on MinorityBarPassage:Bridging
the Gap Between Theory and Practice, 16 T. Marshall L. Rev. 419 (1991)
(examining bar passage rates of African-American, Native American and
Hispanic applicants); W. Sherman Rogers, The ADA, Title VII, and the Bar
Examination: The Nature and Extent of the ADA's Coverage of Bar Examina-
tions and an Analysis of the Applicability of Title VII to Such Tests, 36 How.
L.J. 1 (1993) (analyzing discriminatory administration of bar exams against
disabled applicants); Arthur E. Ryman, Jr., Women and the Bar Exam:
Thinking Like A Woman Lawyer, 37 Drake L. Rev. 79 (1987-88) (examining
problems of female applicants with bar exams). See also J.C. Barden,
Chargesof Sex Bias Put Exam Processon Trial, N.Y. Times, July 22, 1988, at
B20 (reporting that the New York State Board of Law Examiners released a
statement affirming its commitment to gender-neutral grading of bar exams
after students in a bar review course claimed their instructor suggested
otherwise).

4. Claim that Bar Examiners Are Intentionally Keeping Scores Low


Smith's claim that the bar examiners are keeping the pass rate artificially
low, in order to keep lawyer incomes, artificially high, also is likely to fail.
Some studies seem to support Smith's claim. See, e.g., B. Peter Pashigian,
1996] ANECDOTAL HISTORY OF THE BAR EXAM

OccupationalLicensing and the Interstate Mobility of Professionals, 22 J.L. &


Econ. 1, 24 (1979) ("Occupational licensing has had a quantitatively large
effect in reducing the interstate mobility of professionals."). Other studies,
however, have come to an opposite conclusion. See, e.g., Malcolm Getz et
al., Competition at the Bar: The CorrelationBetween the BarExaminationPass
Rate and the Profitabilityof Practice, 67 Va. L. Rev. 863, 869 (1981) (casting
doubt on previous studies that have found a connection between low bar
exam pass rates and high attorney earnings).
Regardless of which studies one chooses to believe, one thing is clear:
neither federal nor state courts will second-guess bar examiners when it
comes to drawing the line between success and failure on the bar exam. See,
e.g., Hoover v. Ronwin, 466 U.S. 558, 581 (holding that denial of admission to
the bar is state action and therefore is exempt from antitrust laws), reh'g
denied, 467 U.S. 1268 (1984); In re Investigation of Conduct of Examination
for Admission to Practice Law, 33 P.2d 829, 832 (Cal. 1934) (en banc)
(upholding California's system of grading bar exams). In large part this
deference by courts is because most, if not all, qualified applicants eventu-
ally do pass: "Ultimately, the majority of graduates of most schools pass a
bar examination, and at many prestigious law schools the pass rate is
comfortably high." Charles W. Wolfram, Modem Legal Ethics § 5.3, at 199
(1986) (footnote omitted).
The fact that each state is free to set its own passing score has led to a
certain amount of disparity among jurisdictions. It also has encouraged
marginal applicants to look for states with "easy" bar exams (and to avoid
those thought to have difficult exams). For many years, Pennsylvania was
the clear examination of choice because of its lenient grading procedures.
See Frank Reeves, Pennsylvania's 'Easy'BarExam Will Soon Become Harder
to Pass, Pittsburgh Post-Gazette, July 31, 1994, at Al (reporting that
Pennsylvania was once called "the Tijuana of the bar admissions world"). In
order to combat the perception that it was soft on testing, Pennsylvania
changed its grading system beginning with the February 1995 bar exam. Id.
The result was a dramatic drop in the number of successful candidates. See
Samuel B. Fineman, Bar Exam Pass Rate Declines; First Time Given Under
New Rules, Legal Intelligencer, May 18, 1995, at 1 (reporting that the
February 1995 pass rate was a mere forty-eight percent).
Finally, even if a court were disposed to look favorably on Smith's claim,
her timing could not be worse: in July 1994, a greater percentage of bar
applicants passed the bar exam nationwide than at any time since at least
1969. See Ken Myers, Based On Results of Bar Exams, Class of 1994 is One of
Brightest, Nat'l L.J., Jan. 9, 1995, at A17 (explaining high pass rates as result
of "test-takers' academic excellence").
GEORGETOWN JOURNAL OF LEGAL ETHICS [Vol. 9:359

C.
Despite his past problems, North is likely to be admitted to the bar.
States traditionally have required bar applicants to demonstrate that they
possess "good moral character." See Deborah L. Rhode, Moral Characteras
a Professional Credential, 94 Yale L.J. 491, 496-503 (1985) (providing
historical overview of states' moral character requirements for bar appli-
cants). Such requirements have consistently withstood challenge both be-
cause of their antiquity and because of the need to ensure that the bar is
comprised of persons who can fulfill their duties as "officers of the court."
See In re Willis, 215 S.E.2d 771, 779 (N.C.), appeal dismissed sub nom., Willis
v. North CarolinaBd. of Law Examiners, 423 U.S. 976 (1975) ("A State can
require high standards for admission to the bar, including good moral
character and proficiency in the laws, so long as the qualifying standards
have a rational connection with the applicant's fitness or capacity to practice
law."); Konigsbergv. State Barof California, 353 U.S. 252, 262-63 ("The term
'good moral character' has long been used as a qualification for membership
in the Bar and has served a useful purpose in this respect."), reh'g denied,
354 U.S. 927 (1957). See also Ken Myers, Deans, Profs Are Held Immune
From Fitness-ReportLawsuits, Nat'l L.J., May 1, 1995, at A18 (reporting on a
ruling that law school officials who file negative recommendations about
students to boards of bar examiners have absolute immunity from liability);
Michael D. White, Comment, Good Moral Characterand Admission to the
Bar: A ConstitutionallyInvalid Standard?, 48 U. Cinn. L. Rev. 876, 879-80
(1979) (noting that the interest of states in regulating membership in their
bars has never been challenged, and calling for constitutionally-precise
standards to prevent arbitrary and inconsistent results).
In most states, the state board of bar examiners has the National
Conference of Bar Examiners conduct a computer background check on
each candidate. Unless this check (which is supplemented by references
from persons supplied by the candidate) turns up information that requires
further investigation, the candidate will be deemed to have the necessary
character. In a few states, the examiners try to go further (usually by
scheduling an interview with the candidate). The most elaborate investiga-
tion system exists in Florida, where applicants are required to register with
the bar examiners while they are still in law school and a multi-million dollar
budget (generated by applicant fees) funds a staff of full-time investigators.
For a further discussion, see Richard L. Abel, American Lawyers 69-71
(1989) (pointing out that even in states like Florida, few candidates are ever
denied admission based on character, and that nationwide the number of
candidates rejected solely on character grounds is thought to amount to no
more than two-tenths of one percent of all applicants).
Even though bar examiners are free to inquire into a candidate's charac-
1996] ANECDOTAL HISTORY OF THE BAR EXAM

ter, they may not do so merely to keep out those whose lifestyles or values
differ from their own. See, e.g., Cord v. Gibb, 254 S.E.2d 71, 73 (Va. 1979)
(ordering admission of an unmarried applicant found unfit by the bar
examiners solely because she lived with a man to whom she was not
married). Instead, the inquiry must be rationally designed to determine
whether the applicant currently is fit to represent clients in a competent and
trustworthy manner. See Schware v. Board of Bar Examiners, 353 U.S. 232,
239 (1957) ("A State can require high standards of qualification ... before
it admits an applicant to the bar, but any qualification must have a rational
connection with the applicant's fitness or capacity to practice law."). See
also Baccus v. Karger, 692 F. Supp. 290, 297-98 (S.D.N.Y. 1988) (approving
requirement that applicants be at least twenty-one years old to be eligible to
sit for the bar exam because maturity is an important character trait for
lawyers); FloridaBd. of Bar Examiners re N.R.S., 403 So. 2d 1315, 1317 (Fla.
1981) (finding applicant's homosexuality irrelevant because sexual prefer-
ence has no bearing on fitness to practice law); In reAnonymous, 577 N.E.2d
51, 54 (N.Y. 1991) (finding applicant did not lack necessary character simply
because he allowed twenty-seven years to go by between the time he passed
the bar exam and the time he decided to apply for admission).
Turning to North's case, it is important to note that, at least to this point,
he has been entirely forthcoming with the bar examiners. As has been noted
elsewhere, lack of candor in the application process "can be deadly." See
Stephen Gillers, Regulation of Lawyers: Problemsof Law and Ethics 631 (4th
ed. 1995) (citing cases); Model Rules of ProfessionalConduct Rule 8.1 (1995)
(suggesting that bar applicants must be truthful). Thus, North is off to a
good start.

1. Drug Abuse
An applicant will not be denied admission simply because his or her past
includes incidents of substance abuse. See, e.g., Application of Strait, 577
A.2d 149, 157 (N.J. 1990) (admitting applicant with history of drug abuse).
It is true, however, that such applicants bear a heavy burden: they must
acknowledge that they had a problem, demonstrate that they are no longer
dependent and show that they are taking steps to avoid returning to their
prior habits. See, e.g., In re Crossley, 839 S.W.2d 1, 5 (Ark. 1992) (denying
admission where applicant had not been drug free for an adequate period of
time); In re Matt, 829 P.2d 625, 630-31 (Mont. 1992) (denying admission
because applicant minimized his role in past drug transactions in represen-
tations to State Bar Committee on Character and Fitness).
The facts state that North has not used drugs in ten years and now acts as
a counselor to others. As such, it appears that North has faced his problem,
solved it and is unlikely to return to it. Thus, North should be admitted.
GEORGETOWN JOURNAL OF LEGAL ETHICS [Vol. 9:359

2. Psychiatric Assistance
Applicants cannot be denied admission simply because they have under-
gone or are undergoing psychiatric counseling. See Campbell v. Greisberger,
865 F. Supp. 115, 121 (W.D.N.Y. 1994) (finding that mental illness cannot
be aperse reason for denial of admission to the New York state bar). It has
been argued by some that the Americans with Disabilities Act (ADA) of
1990 now makes it illegal to even ask about such counseling. See Phyllis
Coleman & Ronald A. Shellow, Ask About Conduct, Not Mental Illness: A
Proposalfor Bar Examiners and Medical Boards to Comply with the ADA and
Constitution, 20 Notre Dame J. Legis. 147, 162-63 (1994) ("Questions about
treatment for mental illness and substance abuse on licensing applications
simply cannot withstand this scrutiny [under the ADA].... Because these
questions discriminate based on disability, they must be eliminated."). In
response to such arguments, at least eight states have modified their bar
applications. Clark v. Virginia Bd. of Bar Examiners, 880 F. Supp. 430, 440
(E.D. Va. 1995).
Of course, even with the passage of the ADA, it seems clear that
applicants who obviously need counseling, but refuse to obtain it, can be
denied admission if they are presently unfit to practice law. See, e.g., In re
Bower, 605 N.E.2d 6, 8 (Ohio 1992) (denying admission to applicant whose
long, rambling answers, coupled with past involuntary commitment and
current refusal to seek treatment, indicated that she would be unable to
function as an attorney); In re Ronwin, 555 P.2d 315, 316 (Ariz. 1976) (en
banc) (denying admission to applicant who suffered from an established
personality disorder because his condition was likely to cause him to bring
and prosecute groundless claims), cert. denied sub nom., Ronwin v. Supreme
Courtof Arizona, 430 U.S. 907 (1977), cert. denied, 439 U.S. 828 (1978).
North, of course, currently is in therapy. Thus, to the extent that he
requires help, he has recognized his need and has taken appropriate action.
At the same time, however, North's condition does not appear to be of a
type that is likely to impair his ability to work as an attorney. Indeed, North
successfully completed law school and passed the bar exam while in therapy.
Thus, North should be admitted despite the fact that he is in treatment.

3. Bankruptcy
Because federal law permits debtors to discharge their debts through
bankruptcy proceedings, coutts have held that bar candidates cannot be
denied admission simply because they choose to avail themselves of the
opportunity to "start over." See, e.g., Kwasnik v., State Barof California, 791
P.2d 319, 327 (Cal. 1990) (admitting applicant who filed for bankruptcy to
avoid satisfying a wrongful death judgment); FloridaBd. of BarExaminers re
1996] ANECDOTAL HISTORY OF THE BAR EXAM

Groot, 365 So. 2d 164, 167-68 (Fla. 1978) (admitting applicant who filed for
bankruptcy to escape having to repay student loans).
North's situation is complicated by the fact that he has filed for bank-
ruptcy twice. Some courts have held that applicants who demonstrate
repeatedly that they cannot handle their personal finances are not entitled
to admission (due to the fact that lawyers are regularly called upon to hold
client funds). See, e.g., In reAnonymous, 549 N.E.2d 472, 473-74 (N.Y. 1989)
(denying admission to applicant who was found lacking in "the character
necessary to discipline himself to control his standard of living and the
amount of his indebtedness, thus showing a lack of financial responsibility
necessary for an attorney"); In re Gahan, 279 N.W.2d 826, 831 (Minn. 1979)
(denying admission to otherwise qualified bar applicant who had defaulted
on important financial obligations). Thus, North will have to show that,
despite declaring bankruptcy twice, these incidents are not part of a larger
pattern of fiscal irresponsibility. For a further discussion, see Alexandra
Stevens, Law School Graduates Who DeclareBankruptcy: Unfit forAdmission
to the Bar?, B. Examiner, Aug. 1993, at 11 (examining current bar admission
policies with respect to applicants who have filed bankruptcy petitions to
discharge debts incurred while acquiring their law degrees).

4. Alleged Sexual Abuse


Although criminal conduct at one time served to automatically disqualify
an applicant, today most states treat it as simply another factor to be
considered in light of the applicant's entire record. See generally Maureen
M. Carr, Note, The Effect of Prior Criminal Conduct on the Admission to
PracticeLaw: The Move to More Flexible Standards, 8 Geo. J. Legal Ethics
367 (1995) (discussing the noticeable shift in recent years away from
disqualification of applicants with criminal records). Nevertheless, cases do
exist in which applicants have been denied admission due to past acts of
sexual abuse or harassment. See, e.g., In re Bellino, 478 N.W.2d 507, 509
(Minn. 1991) (per curiam) (denying admission to applicant who had previ-
ously admitted to sexually assaulting clients); cf. In re Spurling, 595 A.2d
1062, 1063 (Me. 1991) (per curiam) (determining whether applicant should
be denied admission to bar because of his termination from a job for sexual
harassment of female employees).
North's situation, however, is fundamentally different from the facts in
both Bellino and Spurling. First, the charges against North were filed "many
years ago," whereas the charges in both Bellino and Spurling were still fresh
when admission was sought. Second, the charges against North were
dropped prior to trial. In Bellino, they resulted in a conviction and dismissal
from the military, while in Spurling they led to dismissal from a job.
Lack of a criminal conviction is not considered determinative in bar
412 GEORGETOWN JOURNAL OF LEGAL ETHICS [Vol. 9:359

application proceedings. See In re Parker, 838 P.2d 54, 58 (Or. 1992)


("conviction is not a condition precedent" to an applicant's disqualification
from the practice of law "based on that person's dishonest conduct"), cert.
denied sub nor., Parker v. Oregon State Bar, 113 S. Ct. 2440 (1993), cert.
denied, 114 S. Ct. 929, and reh'g denied, 114 S. Ct. 1342 (1994). Here,
however, the fact that the charges were dropped, coupled with the passage
of time and the absence of any subsequent incidents, should lead to North's
admission.

You might also like